Grab Bags!! Flashcards

1
Q

What is BMI

A

kg divided by square of patient’s height in meters

How well did you know this?
1
Not at all
2
3
4
5
Perfectly
2
Q

Is it important to know what liposuction technique is planned?

A

Yes b/c different morbidity and mortality for different techniques
tumescent: large volumes of lidocaine and epinephrine are injected into subcut tissues. Fat removed is less than 3000 mL

semitumescent: more fat is removed and it is associated with fluid overlaod, pulmonary edema, local anesthetic toxicity and fat emboli

How well did you know this?
1
Not at all
2
3
4
5
Perfectly
3
Q

Treatment of nerve injuries :

Risks for nerve injury:

A

usually resolve within 5 days, but ALWAYS seek neurologic consultation.

Extremes in weight
DM
Pre-existing neuro dysfunction
certain positions: lithotomy-common peroneal nerve

How well did you know this?
1
Not at all
2
3
4
5
Perfectly
4
Q

Toxic dose of lidocaine: How much is usally given during liposuction:
How can you decrease risks of local anesthetic toxicity?
Pts at increased risk of lido tox?

A

While the maximum safe dose of lidocaine with epinephrine is often
reported as 7 mg/kg, Dermatology recs: mac dose of 55 mg/kg of lido alone using diluted tumescent solution, adding epinephrine, and
limiting the surgery to less than 3000 ml of fat removal. Moreover, I would ensure
the availability a lipid rescue kit and carefully monitor the patient for signs and
symptoms of local anesthetic toxicity (patients should be monitored throughout the
perioperative period and for at least 30 minutes postoperatively since signs and
symptoms of toxicity may be delayed for over 15 minutes following tumescent
procedures).

pts at increased risk:
extremes of age, cardiac disease, renal dysfxn

How well did you know this?
1
Not at all
2
3
4
5
Perfectly
5
Q

IV fluids during lipo:

A

be cautious, use foley to better balance recognizing that there may be intravascular fluid overload, pulmonary edema or CHF

How well did you know this?
1
Not at all
2
3
4
5
Perfectly
6
Q

Treatment of cancer pain: pt already on morphine. Having inadequate pain control, nausea, vomiting and constipation-what do you recommend?

A

antidepressants and anticonvulsants for neuropathic pain; corticosteroids to reduce
inflammation; bisphosphonates and calcitonin for bone pain; and octreotide to relieve
pain due to bowel obstruction. ANd celiac plexus block

How well did you know this?
1
Not at all
2
3
4
5
Perfectly
7
Q

How would you perform celiac plexus block?

A

prone position, at the level of the Ll vertebral body. Needles are placed approximately 5-7
cm lateral to the midline, and advanced under :fluoroscopic guidance to lie anterior to
the vertebral body.A test block is usually performed with local anesthetic to ensure
benefit from the procedure. If the patient achieves good pain relief from the local
anesthetic, a neurolytic block, with either alcohol or phenol, is then performed.

How well did you know this?
1
Not at all
2
3
4
5
Perfectly
8
Q

complications of celiac plexus block:

A

most serious complication is paralysis, due to spread of the neurolytic
agent into the spinal or epidural space, or secondary to damage of vital arterial supply
to the spinal cord (i.e., the artery of Adamkiewicz). Other complications include
postural hypotension (most common), accidental intravascular injection,
retroperitoneal hemorrhage, sexual dysfunction; pneumothorax, diarrhea, and damage
to the kidneys or pancreas. A celiac plexus block is contraindicated in the presence of
systemic anticoagulation, sepsis, local infection, or bowel obstruction.

How well did you know this?
1
Not at all
2
3
4
5
Perfectly
9
Q

Gabapentin works how?

A

increases GABA in brain (inhibitory neurotransmitte)

How well did you know this?
1
Not at all
2
3
4
5
Perfectly
10
Q

Celiac plexus nerves:

A

T5-T12

How well did you know this?
1
Not at all
2
3
4
5
Perfectly
11
Q

A 35-yr-old man has diffuse burning pain in his left arm that began 6 months ago
after suffering blunt trauma to his hand during a pick-up basketball game. His left
finger tips are cyanotic.

1) What is your differential diagnosis?

A

complex regional
pain syndrome type soft tissue injury, nerve injury (brachia! plexopathy),
vascular insufficiency (Raynaud disease), peripheral neuropathy, or nerve entrapment
syndrome (carpal tunnel syndrome).

How well did you know this?
1
Not at all
2
3
4
5
Perfectly
12
Q

Diagnostic criteria of CRPS-1

Diagnostic testing?

A

initiating noxious event,
followed by burning pain; allodynia or hyperalgesia disproportionate to the degree
and type of injury; cyanosis; edema; cutaneous vasomotor instability (changes in
blood flow); and sudomotor instability (sweating).
With time-smooth and glosy skin, stiff painful joings and NO OTHER CAUSE FOR PAIN OR DYSFUNCTION
Diagnostic testing that may help identify: thermography (to detect vasomotor instability),
sweat testing (to detect sudomotor instability), and radiography (to detect bone
demineralization).

How well did you know this?
1
Not at all
2
3
4
5
Perfectly
13
Q

CRPS 2 vs CRPS 1:

A

CRPS-2 is different from CRPS-1 only in the nature of the inciting
event. Differentiating the two syndromes, therefore, requires a careful history. Some
of the events that can lead to the development of CRPS-1 include, crush injuries,
lacerations, fractures, surgery, sprains, or burns. CRPS-2, on the other hand,
develops following nerve injury, with the characteristic symptoms not necessarily
limited to the distribution of the injured nerve.

How well did you know this?
1
Not at all
2
3
4
5
Perfectly
14
Q

JW-what to do first:

A

Identify what they will accept, we will respect religious beliefs (until-in kids-it is absolutley necessary), Mention you will have to seek a court order authorizing the admin of those blood products

How well did you know this?
1
Not at all
2
3
4
5
Perfectly
15
Q

Pedi Pt crying and says they don’t want surgery:

A

talk to pt and family about why. If due to anxiety, consider removing pt from periop area, discuss giving midaz with pt and family. If it still escalates, have a discussion with surgeon about possible rescheduling

How well did you know this?
1
Not at all
2
3
4
5
Perfectly
16
Q

What is SLE?

A

SLE is an autoimmune disease resulting in systemic chronic inflammation
(i.e. vasculitis) and tissue damage. Diagnosis is often difficult and is usually made based on
the presence of 3 or more of the following criteria: (1) antinuclear antibodies; (2)
characteristic rash (i.e. malar rash and/or discoid rash); (3) nephritis; ( 4) polyarthritis
(symmetrical arthritis involving the hands, wrists, elbows, knees, and/or ankles); (5)
hematologic disorder (i.e. thrombocytopenia, hemolytic anemia, etc.); (6) serositis (i.e.
pericarditis and/or pleuritis); (7) neurologic disorder (i.e. seizures and/or psychosis); and (8)
photosensitivity.

How well did you know this?
1
Not at all
2
3
4
5
Perfectly
17
Q

Pt has lupus anticoagulant-you doing neuraxial?

A

Lupus anticoagulant is a misnomer because this immunoglobulin does not
result in clinical coagulopathy but, rather, is a prothrombotic agent that only causes a
prolonged aPTT because of a laboratory artifact.
So as long as other labs wnl, (of course consider clinical course KIM that lupus. can cause thromboyctopenia jsut like PET)

How well did you know this?
1
Not at all
2
3
4
5
Perfectly
18
Q

Antiphospholipid syndrome: Associated with what?

What lab would you see? Increased risk of bleeding? Seizures in these pts?

A

acquired autoimmune disorder characterized by venous and or arterial thrombosis. CAn happen with lupus or RA or in isolation. prolongation of PTT, but still no increased risk of bleeding. Consider this diagnosis in pts with isolated PTT
Seizures could be due to cerebral embolsim. keep this in mind with pregnat pts who have it and PET

How well did you know this?
1
Not at all
2
3
4
5
Perfectly
19
Q

14 yo pt preggo test comes back positive-wyd?

A

I would first attempt to determine whether my state law would
declare her emancipated in regards to medical decisions involving her pregnancy,
recognizing that this would confer upon her the right to complete confidentiality.
I would most likely inform only the
minor of her positive pregnancy test, encourage her to make her mother aware of the
pregnancy, and attempt to facilitate the appropriate follow up care for the patient (i.e.
obstetrician and social worker)- in order to maintain the trust in the physician-patient relationship

How well did you know this?
1
Not at all
2
3
4
5
Perfectly
20
Q

Concerns about anesthesia for MRI?

A

(1) the unintentional transfer of a
ferromagnetic object (i.e. gas cylinders, keys, scissors, etc.) into the scanner room, leading to
projectile-induced injury to the patient or hospital personnel; (2) dislodgement or
malfunction of an implantable device with exposure to the scanner’s magnetic field (i.e.
pacemaker, AICD, implanted infusion pump, spinal cord stimulator, and/or mechanical heart
valves); (3) magnet-induced equipment malfunction (i.e. monitors and infusion pumps); (4)
thermal injury, secondary to magnetic field affects on monitoring equipment like ECG pads
or the pulse oximeter (monitoring cables should be straight where in contact with the
patient. .. i.e. no coiling); (5) temporary or permanent hearing loss secondary to the loud
banging produced by the MRI scanner (ear plugs should be utilized to prevent this type of
injury); (6) patient anxiety,

How well did you know this?
1
Not at all
2
3
4
5
Perfectly
21
Q

Would you do an MRI in preggo? Braces in preggo?

A

The evidence does not currently suggest that the magnetic fields generated
during MRI are harmful to the baby in utero,

While the presence of braces is not a contraindication to MRI, they could
potentially degrade the quality of the image. Therefore, I would discuss this with the
radiologist to determine if MRI is still the optimum modality for this patient. If I had any
doubt about the safety of an object, I would use a small hand-held magnet to test whether the
object was ferromagnetic prior to entering the scanner room.

How well did you know this?
1
Not at all
2
3
4
5
Perfectly
22
Q

How will you give a pt anesthesia in MRI if you do NOT have an MRI compatible machine or monitors?

A

in the absence of an MRI-compatible anesthesia machine, I would: (1) ensure that
an Ambu bag was connected to an oxygen source in the scanner room; (2) apply the
appropriate monitors with sufficiently long cables to reach the area just outside the scanner
room; (3) administer a B2-agonist; (3) induce the patient with lidocaine, fentanyl, versed, and
propofol; (4) secure her airway with an ETT, to provide a definitive airway in this obese
asthmatic patient (an LMA may be considered to reduce the risk of bronchospasm, but would
be inferior in the management ofbronchospasm should it occur); and (5) maintain anesthesia
with either propofol or a volatile agent delivered through lengthy tubing that allows the
anesthesia machine and/or infusion pump to remain outside of the scanner room.

How well did you know this?
1
Not at all
2
3
4
5
Perfectly
23
Q

Pt with heart transplant has 2 p waves on EKG:

A

The non-conducted P-wave is unlikely to represent atrioventricular block, but
rather, is originating from residual native atrial tissue. The impulse from the native sinus
node is unable to traverse the anastomotic line and, therefore, does not result in ventricular
contraction and the generation of a QRS complex on ECG.

How well did you know this?
1
Not at all
2
3
4
5
Perfectly
24
Q

Monitors for pt with heart transplant:

A

Pretty much the same, but this is a preload dependent patient because the denervatedheart primarily
increases cardiac output by increasing stroke volume via the Frank-Starling mechanism,
rather than heart rate),
These pts at risk for coronary atherossclerosis, arrhythmias, So make sure you place an A line, and a 5 lead EKG. Consider placement of CVP, but weigh that with risk of infection in transplant patients.

How well did you know this?
1
Not at all
2
3
4
5
Perfectly
25
Q

Frank Starling:

A

increase in SV that results from more forceful cardiac contraction in response to increased cardiac filling. Helpful in transplanted heart physiology

How well did you know this?
1
Not at all
2
3
4
5
Perfectly
26
Q

Nasal intubation in heart transplant pts?

A

increased risk of infection from nasal flora-so no if possible

How well did you know this?
1
Not at all
2
3
4
5
Perfectly
27
Q

Induction in transplanted heart pts:

Good med for pt with transplanted heart?

A

Slow controlled to avoid drops in SVR-use etomidate

Good med: Isoprterenol: potent beta 1 and beta 2 adrenoreceptor agonist-excellent choice in pts with cardiac transplant

How well did you know this?
1
Not at all
2
3
4
5
Perfectly
28
Q

You’re going to reverse a transplanted heart pt with neostigmine: do you need to give glyco?

A

Yes. even though it wont’ change the HR< it can help reduce bronchospasm and increase salivation. KIM that neostigmine can cause bradycardia in transplanted hearts, so be ready

How well did you know this?
1
Not at all
2
3
4
5
Perfectly
29
Q

NSAID post op for transplanted heart pt:

A

If on nephrtoxic immunosuppressive drugs-NO. could exacerbate kidney damage, and also increase risk of gastritis (pts have gastritis with immunosuppressive drugs)

How well did you know this?
1
Not at all
2
3
4
5
Perfectly
30
Q

You’re called to be present for possible sick baby being delivered-what do you need to have ready?

A

Prior to delivery I would ensure that the appropriate support personnel
and equipment were available for neonatal resuscitation (i.e. a self-inflating bag
attached to 100% oxygen, a neonatal oxygen mask, wall suction, a radiant warmer or
other heat source, warmed linens, appropriately sized laryngoscopy blade and ETTs,
resuscitation medications, and blow by oxygen capabilities).

How well did you know this?
1
Not at all
2
3
4
5
Perfectly
31
Q

When are you suctioning out meconium?

A

(tracheal suctioning
should be performed if the baby does not demonstrate strong respiratory effort,
good muscle tone, and a heart rate > 100 beats/minute).

How well did you know this?
1
Not at all
2
3
4
5
Perfectly
32
Q

Neonatal resuscitation outline”

A

below 100, he
remained apneic, or he was gasping, I would provide positive mask ventilation with
air, or an air/oxygen mixture, and consider applying a pulse oximeter. If after 30
seconds of positive mask ventilation the HR was less than 60, then I would INTUBATE
increase the oxygen concentration to 100%, begin chest compressions (3
compressions to 1 breath for a total of 120 events/minute), and establish venous(umbilical·vein catheterization).~r intraosseous ac~~~~ (risks include tibial fracture
and osteomyelitis). If after another 30 seconds there were still no improvement, I
would administer 0. 01-0. 03 mg/kg of epinephrine via the umbilical vein or
established intraosseous access, and consider volume expansion.

How well did you know this?
1
Not at all
2
3
4
5
Perfectly
33
Q

Let’s say baby is not doing well due to magnesium toxicity-wyd?
what about hypoglycemia?
what about if baby has mom on opioids?

Why do you have to be careful with calcium?

A

If
magnesium toxicity were confirmed, I would administer calcium (100 mg/kg of
calcium gluconate or 30 mg/kg of CaCh), recognizing that calcium therapy has been
associated with cerebral calcification and decreased survival in stressed newborns
(therefore, it should only be administered to reverse the effects of magnesium
toxicity). In the case of hypoglycemia (glucose< 35 mg/dL), glucose should be
administered (8 mg/kg/minute of 10% solution). While placental transfer of
maternally administered narcotics could potentially be exacerbating this neonate’s
condition, I would not administer naloxone unless all other resuscitative efforts had
failed; as this drug is no longer recommended during the initial resuscitation in the
delivery room (can worsen the neurologic damage caused by asphyxia).

Calcium can cause cerebral calcification and decreased survival in stressed newborns.

How well did you know this?
1
Not at all
2
3
4
5
Perfectly
34
Q

Mallampati score:

A

Class 1: complete visualization of soft paalte

2: complete visualization of the uvula
3: Visualization of only base of uvula
4: soft palate not visible at all

How well did you know this?
1
Not at all
2
3
4
5
Perfectly
35
Q

Laryngoscopy grades:

A

1: Full view of glottis
2a: partial view of glottis
2b: only arytenoids seen
3: only epiglottis seen
4: neither glottis nor epiglottis seen

How well did you know this?
1
Not at all
2
3
4
5
Perfectly
36
Q

What questions do you want to ask the pt with cirrhosis?

A

determine the severity of his hepatic disease by
performing a thorough history and physical, focusing on the onset and etiology of his
cirrhosis, and the presence of jaundice, bleeding disorders, ascites, asterixis, and
hepatic encephalopathy. Based on my findings, I may consider additional lab work to
aid in discerning the severity of liver disease, including bilirubin, transaminases,
alkaline phosphatase, albumin, total protein, prothrombin time, INR, and hepatitis
serologies

How well did you know this?
1
Not at all
2
3
4
5
Perfectly
37
Q

Systemic effects of cirrhosis

A

Cirrhosis’ effects on the respiratory system include intrapulmonary arteriovenous
(AV) shunts, reduced PRC, restrictive lung disease, pleural effusions, and attenuation
of hypoxic pulmonary vasoconstriction. Cerebral effects include the accumulation of
ammonia and other toxins, which may lead to encephalopathy. Thrombocytopenia
and clotting factor deficiencies may result in a coagulopathy. Cardiovascular effects
include decreased peripheral vascular resistance, increased cardiac output, and
cardiomyopathy. Potential metabolic effects include dilutional hyponatremia,
hypokalemia, hypoglycemia, and hypoalbuminemia. Various additional effects of
cirrhosis include portal hypertension, esophageal varices, delayed gastric emptying,
ascites, and hepatorenal syndrome

How well did you know this?
1
Not at all
2
3
4
5
Perfectly
38
Q

Muscle relaxant in cirrhosis:

And what if that choice wasnt’ available?

A

I would prefer to use a muscle relaxant that is not dependent on
hepatic metabolism, such as Cis-Atracurium. This drug’s duration of action should
not be affected by liver failure, because it undergoes degradation in the plasma by
Hofinann elimination, and is reduced to inactive metabolites. If that choice unavailable-i would recognize potential for increased duration of action and monitor nm fxn with nerve stimulator

How well did you know this?
1
Not at all
2
3
4
5
Perfectly
39
Q

Signs of PDPH and what you would order before treatment?

A

(PDPH), I would
consider an epidural blood patch. Initially, however, I would review the patient’s
record and assess him for signs and symptoms of a PDPH, such as frontal-occipital
headache, decreased pain with recumbent positioning, nausea, vomiting, neck
stiffness, back pain, visual disturbances (photophobia, diplopia, and difficulty in
accommodation), and auditory disturbances (tinnitus, hypacusis, and hearing loss). I
would also require a coagulation profile, since he may be receiving anticoagulants
following his recent knee surgery. If a blood patch were contraindicated for any
reason, I would recommend conservative therapy, which consists of hydration,
caffeine, and pain control.

How well did you know this?
1
Not at all
2
3
4
5
Perfectly
40
Q

Why do pts have pain with PDPH?

A

The lost CSF, decreased buoyant support for the brain, and cerebral
vasodilation (increased cerebral blood flow to compensate for decreased CSF) that
accompanies significant dural puncture, can lead to the signs and symptoms of a
PDPH, which include a frontal-occipital headache, decreased pain with recumbent
positioning, nausea, vomiting, neck stiffness, back pain, photophobia, diplopia,
difficulty in accommodation, tinnitus, and hypacusis (hearing loss). Rarely, PDPH is
associated with seizures (most likely secondary to cerebral vasospasm), abdominal
pain, and diarrhea. Loss of CSF can also lead to cranial nerve stretching with
subsequent palsy.

How well did you know this?
1
Not at all
2
3
4
5
Perfectly
41
Q

non blood patch options for PDPH:

A

While less effective than blood patch, conservative treatment
consisting of hydration (no evidence of therapeutic benefit), caffeine, the placement
of an abdominal binder (increases abdominal pressure, possibly leading to an increase
in CSF pressure), and pain control provides an alternative when a blood patch is
contraindicated.

How well did you know this?
1
Not at all
2
3
4
5
Perfectly
42
Q

Type D TEF:

A

esophageal pouch communicates with trachea, but is otherwise the same as c
(esophageal atresia with blind upper pouch and lower segment tracheal fistula)

How well did you know this?
1
Not at all
2
3
4
5
Perfectly
43
Q

Where should ETT be in TEF? How can you do this? What can surgeon help you with?

A

Ideally the tip of the ETT should be placed distal to the fistula and
proximal to the carina, allowing positive pressure ventilation of both lungs without
excessive airflow through the fistula
One method of properly positioning the ETT is to advance the ETT
into the right main-stem bronchus and then slowly withdraw it until breath sounds are
heard through a stethoscope placed in the left axilla.
If excessive airflow through fistula occurs surgeon can perform a gastrostomy to decompress the stomach

How well did you know this?
1
Not at all
2
3
4
5
Perfectly
44
Q

Epiglottitis symptoms:

If pt presents with severe respiratory distress, are you getting radiographs?

A

SUDDEN ONSET, fever, drooling, stridor, substernal retractions. Radiographs should only be done when child is stable and when appropriate people can accompany child. Severe should immediately be transferred to OR with Anesthesiologist and the Surgeon

How well did you know this?
1
Not at all
2
3
4
5
Perfectly
45
Q

Pediatric pt with epiglottitis has a family Hx of MH-wyd?

A

100% O2
Emergency drugs and airway stuff ready
No versed
Parents come with me
stan ASA monitors, emergency airway cart and surgeon in room-an intramuscular dose of ketamine (2-3 mg/kg) with the goals of
providing sedation and maintaining spontaneous ventilation while securing
intravenous access.
Then deepen anesthesia with IV medications, place tube 1/2 size smaller than normal. Let the surgeon do their thing and then consider switching to nasal intubation as it is typically tolerated better in children.

How well did you know this?
1
Not at all
2
3
4
5
Perfectly
46
Q

When would you extubate kid with epiglotkfjd?

A

I would only consider extubation once the child’s fever, neutrophilia,
and epiglottic swelling had resolved (these patients usually remain intubated for 24-
48 hours following the initiation of treatment). The resolution of airway edema is
suspected with the return of swallowing and when a significant leak around the
nasotracheal tube is present (10-20 cm. H20 peak inflation pressure). Prior to
extubation, I would transfer the patient to the operating room, induce general
anesthesia, and confirm the resolution of airway edema by visual inspection with a
flexible fiberoptic bronchoscope. I would then extubate the child and continue to
monitor her for post-extubation edema.

How well did you know this?
1
Not at all
2
3
4
5
Perfectly
47
Q

In pituitary tumors, dont’ forget to ask:

A

What kind is it? Cushings, acromegaly, see if there is suprasellar extension (ha, blurry vision, rhinorrhea)

How well did you know this?
1
Not at all
2
3
4
5
Perfectly
48
Q

During infiltration of local anesthesia, you notice multiple premature ventricular
contractions (PVCs). What would you do? (LA for tumor resection) why do you think it happened?

A

The PVCs are likely to be the result of the injection of epinephrine-
containing local anesthetic or the use of cocaine pledgets. With this in mind, I would

alert the surgeon to stop the infiltration, ensure adequate ventilation with 100%
oxygen, check blood pressure, and examine the EKG for signs of myocardial
ischemia. I would then be prepared to treat any hypertension and/or significant
arrhythmias that may occur.
4) What do you think was the cause?
UBP Answer: As I mentioned, the most likely cause is systemic uptake of
epinephrine contained in the local anesthetic. However, other potential causes
include, hypoxia, cardiac ischemia, air embolism, electrolyte abnormalities, or
anesthetic induced cardiac depression.

How well did you know this?
1
Not at all
2
3
4
5
Perfectly
49
Q

A 23-year-old pregnant patient at 33 weeks gestation, who is receiving terbutaline
for premature labor, presents with respiiratory distress. On exam, you hear crackles
bilaterally. Why do you think she is in distress?

A

The bilateral crackles on exam are suggestive of pulmonary edema,
which could be associated with the use of terbutaline, preeclampsia (increased vascular permeability), pulmonary
embolism, or a previously unrecognized cardiac condition. Although a rare
complication, terbutaline use has been associated with pulmonary edema.

How well did you know this?
1
Not at all
2
3
4
5
Perfectly
50
Q

How would you tx pulmonary edema in the preggo pt? Pt in resp distress

A

It would depend on identifying and treating the cause. cardiogenic in origin (i.e. myocardial ischemia,
cardiomyopathy, or a dysrhythmia), I would consider using diuretics, inotropic
agents, beta-blockers, or afterload reducing agents, as indicated.non-cardiogenic in origin, I might simply provide diuretic
therapy and fluid restriction. In either. case, I would consider providing ventilatory
support with PEEP. While intubation and mechanical ventilation may prove
necessary, I would first consider using CPAP.

How well did you know this?
1
Not at all
2
3
4
5
Perfectly
51
Q

How does PEEP help with pulmonary edema? KIM what with PEEP and cardiac stuff?

A

PEEP improves oxygenation and pulmonary function by redistributing
alveolar fluid to areas that are less involved in gas exchange, and by recruiting
collapsed alveoli that are contributing to pulmonary shunting. KIM that PEEP can worsen cardiac fxn secondary to decreased preload

How well did you know this?
1
Not at all
2
3
4
5
Perfectly
52
Q

Doesn’t PEEP just hyperinflate the already open alveoli and risk barotrauma?

A

While PEEP has the potential to hyperin:flate already open alveoli, the
goal is to achieve optimum PEEP where collapsed alveoli are recruited

How well did you know this?
1
Not at all
2
3
4
5
Perfectly
53
Q

Shortly after receiving a stellate ganglion block with bupivacaine, a 28 year old
female patient is noted to have difficulty speaking and reports dizziness, difficulty
swallowing, and shortness of breath. DDX?

Also-what is a stellate ganglion block used for? where? side effects/risks?

A

LAST
Total spinal (with resp sxs to follow) 1) paralysis of the recurrent laryngeal nerve, which is located near the stellate
ganglion (would result in hoarseness); 2) pneumothorax could lead to respiratory
distress and hypotension (the latter occurring with a tension pneumothorax);

Indications: CPRS. Also angina, phantom limb pain, vascular insufficiency, hyperhidrosis
Anatomy: Inferior cervical + T1 sympathetic ganglia @ C7. Lays under the SCM/carotid, above the lung
Side Effects: Horner’s (intentional), hoarseness (RLN), eleveated hemidiaphragm (phrenic)
Complications: hematoma, brachial plexus injury, pneumothorax, esophageal perforation, intrathecal/epidural/intravascular injection

How well did you know this?
1
Not at all
2
3
4
5
Perfectly
54
Q

If a pt had last, but was not unconscious-tell me how you would proceed? What are you avoiding? When do you make the decision to give intralipid?

A

(1) call for help and a lipid rescue kit; (2) ensure adequate
ventilation and oxygenation to correct and/or avoid factors that enhance the systemic

toxicity of local anesthetics, such as hypercarbia (increased cerebral blood flow, intra-
neuronal ion trapping of the drug, and decreased plasma protein binding of the drug),

acidosis (decreased plasma protein binding of the drug), and hypoxemia; (3)
administer a benzodiazepine to stop the seizure (seizure activity increases
metabolism, which may lead to hypoxemia, hypercarbia, and acidosis); ( 4) administer
succinylcholine and intubate her if ventilation were inadequate, the risk of aspiration
was significant (history of GERD or hiatal hernia), or if tonic-clonic movements
persisted despite benzodiazepine administration (succinylcholine would minimize the
metabolic acidosis associated with seizure-induced.muscle activity, it would not,
however, affect the acidosis that develops secondary to seizure-induced increases in
cerebral metabolism); (5) alert the nearest facility with cardiopulmonary bypass
capability; and ( 6) treat hypotension, bradycardia, and dysrhythmias as indicated
(avoid procainamide, lidocaine, j3-blockers, vasopressin, and calcium channel
blockers when treating bupivacaine-induced cardiac arrhythmias). Moreover, I
would (7) initiate lipid emulsion therapy if the signs and symptoms of local anesthetic
toxicity appeared to be rapidly progressing, she experienced prolonged seizure
activity, or she developed signs of cardiac toxicity (i.e. bradycardia, heart block,
hypotension, asystole, or ventricular arrhythmia). Finally, if the patient did not
respond adequately to these therapies, I would (8) consider utilizing cardiopulmonary
bypass to provide “bridging” therapy until her tissue levels of local anesthetic were
no longer toxic.

How well did you know this?
1
Not at all
2
3
4
5
Perfectly
55
Q

Dosing for last? Can you repeat it? What is the max dose?

A

Bolus 1.5 mL/kg of 20% lipid solution over 1
minute (roughly 100 mL in adults) and start a continuous infusion at 0.25
mL/kg/minute. If cardiovascular instability persists after 5 minutes, repeat the
bolus and double the infusion rate. Maintain the infusion for at least lO·minutes
after attaining circulatory stability. The recommended upper limit for initial lipid
dosing is approximately 10 mL/kg over 30 minutes.

How well did you know this?
1
Not at all
2
3
4
5
Perfectly
56
Q

DON’T FORGET TO CALL FOR WHAT IN EMERGENT SITUATIONS?

A

HELP!

How well did you know this?
1
Not at all
2
3
4
5
Perfectly
57
Q

LAST pt progresses to asystole-now what? What do you give less of in LAST?

A

I would immediately: (1) start chest compressions, (2) call for a
defibrillator (in case a shockable rhythm developed during resuscitation), (3) secure
the airway with an endotracheal tube, and ( 4) provide 100% oxygen. Next, I would
(5) attempt to confirm true asystole (make sure all cables are connected properly,ensure adequate monitor gau;, check another lead, ‘check for pulse - very fine
ventricular fibrillation can look like asystole), (6) initiate lipid emulsion therapy (if
not already done), (7) administer a 1 μg/k:g intravenous bolus of epinephrine,
recognizing that higher doses of epinephrine have been associated with poorer
outcomes in the setting ofbupivacaine-induced asystole (epinephrine is highly
arrhythmogenic and may reduce the efficacy of lipid emulsion therapy), and (8)
correct any potential contributing factors, such as hypoxia, hypercarbia, and acidosis.
I would then (9) continue to monitor the patient for the development of a shockable
rhythm, (10) notify the nearest facility with cardiopulmonary bypass capabilities, and
(11) consider placing her on bypass if she remained inadequately responsive to these
interventions.
Continue CPR for at least 60 min

How well did you know this?
1
Not at all
2
3
4
5
Perfectly
58
Q

If Epi isn’t working in last-you giving vasopressin?

A

NO!!! No vasopressin in LAST

How well did you know this?
1
Not at all
2
3
4
5
Perfectly
59
Q

What are the most important factors affecting systemic absorption of local
anesthetic?

What determines onset of local anesthetics? How can you make this faster?

What determines amount of systemic absorption?

A

the amount of blood flow at the site of injection (intravenous>
tracheal > intercostal > caudal > paracervical > epidural > brachial plexus >
sciatic/femoral> subcutaneous), (2) the dose, (3) the properties of the injected local
anesthetic (the higher the lipid solubility and protein binding of a drug, the lower the
rate of systemic absorption), and ( 4) the addition of vasoconstrictors to the local
anesthetic solution (the vasoconstriction associated with the addition of epinephrine
decreases the rate of systemic absorption).

The closer to physiologic pH that the pKa is, the more anesthetic exists in the uncharged base form. With the exception of benzocaine, the pKa’s of all local anesthetics are greater than tissue pH (7.4). Lower pKa results in a more rapid onset. Onset is also reduced by increasing the solution pH.

Onset can also be shortened by increasing the concentration or dose. Chloroprocaine, despite having a pKa of 9, has a rapid onset because of its use in high (as high as 3%) concentrations.

More lipophilic and protein bound a local anesthetic is, less systemic absorption

How well did you know this?
1
Not at all
2
3
4
5
Perfectly
60
Q

Ways to prevent LAST?

A

limit the administered dose of
local anesthetic drug,Other preventative measures include the use of
ultrasound, aspiration prior to injection, the use of small incremental drug dosing, avoidance of excessive sedation during block

How well did you know this?
1
Not at all
2
3
4
5
Perfectly
61
Q

Heart transplant pt has 2 p waves-why? will this affect conduction?

A

The ECG of patients with a transplanted heart often shows two sets of
P-waves, with the origin of one P-wave being the native sinoatrial node, while the
origin of the other is the donor’s sinoatrial node. While the native sinoatrial node
may continue to be affected by autonomic influences, it does not alter cardiac
function because the generated impulse is unable to traverse the suture line between
the native heart and transplanted heart

How well did you know this?
1
Not at all
2
3
4
5
Perfectly
62
Q

Pregnant lady has decreased in FHR immediately after placement of spinal analgesia. Why? And what are some other causes? When is it more pronounced? Does it lead to increase rate of c section?

A

The onset of fetal bradycardia following the initiation of spinal
analgesia suggests that the (1) rapid onset of analgesia has resulted in an abrupt
decrease in circulating epinephrine, with subsequent uterine hypertonus .
(tachysystole) and decreased uterine perfusion (spinal anesthesia –> rapid decrease in
pain —> abrupt decrease in plasma epinephrine - decreased stimulation of beta-
adrenergic receptors in the uterus —> increased uterine tone —> reduced uterine blood
flow (most uterine blood flow occurs during uterine relaxation/diastole) —>
diminished fetal oxygen delivery —-> fetal bradycardia).

other less likely causes: maternal hypotension secondary to sympathectomy (spinal
anesthesia), aortocaval compression (inadequate left uterine displacement),
hypovolemia

The increase in uterine tone that can occur with the initiation of neuraxial
anesthesia to control labor pain is more pronounced when: (1) oxytocin is utilized,
(2) spinal anesthesia is employed rather than epidural (more rapid
sympathectomy), (3) the height of sensory blockade is relatively higher, and (4)
when there is a greater difference between pre and post-analgesia pain scores.
• The, fetal bradycardia following neuraxial anesthesia is usually transient and
readily treatable. Moreover, it does not increase the incidence of emergency
cesarean section or the overall risk for adverse outcome.

How well did you know this?
1
Not at all
2
3
4
5
Perfectly
63
Q

What would you do in this transplanted heart pt with decreased FHR after spinal? What’s especially important to this pt?

A

In addressing the fetal bradycardia, I would: (1) provide supplemental
oxygen, (2) ensure adequate left uterine displacement (especially important in this
patient with a transplanted heart, due to the dependence on preload in responding to
hypotension), (3) discontinue any intravenous oxytocin administration, (3) treat any
maternal hypotension, and ( 4) consider fetal scalp stimulation. If the uterine
hypertonus and fetal bradycardia persisted, I would (5) administer a tocolytic (i.e. 50-
100 μg of intravenous nitroglycerine or 125-250 μg of intravenous terbutaline (careful-it can cause pulmonary edema)

64
Q

Heart transplant pt has to go for C section due to PeT: what are you going to check/evaluate prior to? (assuming not emergent)

A

In evaluating this patient, I would: (1) talk to the transplant team about
the patient’s cardiac status, irnrnunosuppression, and the need for antibiotic
prophylaxis; (2) review the most recent echocardiography, endomyocardial biopsy,
and/or coronary angiography results, and evaluate the patient for signs of rejection
such as arrhythmias, fever, malaise, shortness of breath, accelerated coronary
atherosclerosis, and myocardial dysfunction (when there are signs of rejection,
echocardiography, coronary angiography, and endomyocardial biopsy should be
repeated as time permits); (3) identify and evaluate any pacemaker placed to control
bradydysrhythrnias (many post-cardiac transplant patients require a pacemaker for
this purpose); ( 4) determine whether her hypertension is due to preeclampsia or is
chronic in nature, and treat accordingly ( cyclosporine-induced hypertension is
common in this patient population); ( 5) evaluate her ECG for signs of myocardial
infarction and/or ischemia, recognizing that cardiac denervation masks the usual
symptoms associated with these events (i.e. silent ischemia); (6) identify any
complications associated with irnrnunosuppressive therapy, such as nephrotoxicity
(secondary to cyclosporine therapy), bone marrow suppression, osteoporosis,
hepatotoxicity, and opportunistic infection; (7) administer a stress dose of
corticosteroid, if not already done (these patients are usually receiving steroids as part
of their irnrnunosuppressive therapy); (8) ensure norrnovolemia, recognizing that this
preload dependent patient (transplanted heart) is likely hypovolemic (secondary to PET and HTN), obtain platelet level (PET), examine airway.

65
Q

Transplant pts: is preggo well tolerated? What can they develop and why?

A

97% of patients develop hypertension by 10 years post-transplant. This is often
attributed to treatment with cyclosporine. Treatment usually involves the use of a
calcium channel blocker (i.e. diltiazem) and/or an ACE inhibitor. Nifedipine may
be a less desirable choice since it may cause significant vasodilation, which is
poorly tolerated by these patients who are preload dependent. !)-blockers should
also be avoided, if possible, since cardiac responsiveness during exercise is
dependent of circulating catecholamines.
• Allograft rejection usually occurs in the first 6 months and presents with fever,
malaise, arrhythmias, shortness of breath, accelerated coronary atherosclerosis,
and myocardial dysfunction. Hemodynamic compromise is considered a late sign
of rejection. Rejection is typically treated with steroids, but may require
immunosuppression with IV immunoglobulin or plasmapheresis .
• Immunosuppressive therapy usually includes cyclosporine, azathioprine, and
prednisone.
• Pregnancy is usually well tolerated by post-cardiac transplant patients.
• In the setting of renal dysfunction, avoid drugs that may exacerbate renal
dysfunction (i.e. NSAIDs) or primarily undergo renal excretion.

66
Q

Anesthesia for a pregnant woman with a transplanted heart?

A

Assuming there were no signs of coagulopathy (increased risk in the
setting of preeclampsia ), I would utilize her epidural catheter for the case in order to
avoid the instrumenting the airway of this pregnant and preeclamptic patient who is at
increased risk for difficult airway management. However, I recognize that
immunosuppression therapy increases the risk for infection, and denervation makes
the transplanted heart dependent on circulating catecholamines and preload

(autonomic influences are absent, but the Starling relationship between diastolic end-
diastolic volume and cardiac output is normal). Therefore, I would pay special

attention to aseptic technique and raise the levels of epidural analgesia very slowly to
avoid a rapid sympathectomy that would potentially reduce preload (the preloaddependence associated with the transplanted heart makes epidural anesthesia
preferable to spinal anesthesia).

General, spinal and epidural can all be done-just be careful with preload

67
Q

Nasal intubation in pt with transplanted heart? Central lines?

A

Due to the increased risk of infection:
o Invasive monitoring and lines should only be employed when necessary
o Oropharyngeal intubation is preferred to the nasopharyngeal route (the
latter is associated with an increased risk of infection due to the organisms
typically located in the nasopharynx).

68
Q

You decide to proceed with general anesthesia. Just after induction the patient
experiences a significant drop in blood pressure. What do you think is going on?
Other DDX:

A

Given the timing of her hypotension, the most likely etiology is (1) an
anesthetic-induced drop in systemic vascular resistance, with a subsequent reduction
in preload. Since the transplanted heart is initially unable to respond to acute
hypotension with an increase in heart rate due to the lack of autonomic innervation, it
relies on adequate preload to maintain cardiac output (since a and Beta receptors remain
intact on the transplanted heart, there is a delayed increase in heart rate in response to
increased circulating catecholamines).

other:anaphylaxis, pe, Adrenal suppression (most cardiac pts on steroids)

69
Q

Transplanted heart stuff:
Response to circulating catecholamines
HR response to carotid sinus, oculocariac:
What causes their high heart rate?

A

The response to circulating catecholamines is normal and possibly enhanced due
to denervation sensitivity (increased receptor density), even though sympathetic
innervation is absent.
• The heart rate response to normal respiratory variations (sinus arrhythmia),
carotid sinus massage, valsalva maneuvers, and the oculocardiac reflex are absent.
• The absence of vagal influences causes a relatively high resting heart rate (90-120
beats/minute)

70
Q

How would you treat hypotension in this transplanted heart pt who is preggo? Which vasoactive agent would you choose and why?

A

In treating her hypotension, I would: (1) ensure adequate left uterine
displacement; (2) evaluate her ECG for signs of arrhythmia and/or myocardial
ischemia; (3) administer fluids to maintain high-normal preload, keeping in mind that
overaggressive fluid may lead to pulmonary or cerebral edema in this patient with
severe preeclampsia; (4) administer a direct vasoconstrictor, such as phenylephrine
(indirect vasopressors, such as ephedrine, are not as effective due to the absence of
catecholamine stores in myocardial neurons); and (5) reduce or discontinue any
volatile agent. If these interventions are ineffective, I would (6) consider starting an
isoproterenol or epinephrine infusion and (7) evaluate the patient for signs of
anaphylaxis (i.e. hives, flushing, swelling, and bronchospasm) or pulmonary
embolism (thrombotic or amniotic fluid).

71
Q

Preggo heart transplant pt: It becomes necessary to administer a nondepolarizing muscle relaxant_ during
the procedure and you are preparing to reverse muscle relaxation at the end of
the case. Is it necessary to administer a muscarinic agonist along with the
cholinesterase inhibitor in the setting of a denervated heart?

A

I would administer a muscarinic antagonist along with the
cholinesterase inhibitor for reversal of muscle relaxation, since some cardiac
reinnervation may occur after 6 months post-transplant. In addition to that, it has benefits of decreased saliva

72
Q

Patient has MVP but is on no medications-does she need further work up? (she is scheduled for shoulder surgery in the sitting position)

A

The presence of asymptomatic mitral valve prolapse uncomplicated by
other medical conditions is not a sufficient reason for further cardiac testing.
However, ifher MVP were associated with significant mitral regurgitation, syncope,
chest pain, or symptoms of congestive heart failure, further testing may be warranted.
Therefore, I would begin by performing a focused history and physical exam to illicit
signs and symptoms of congestive heart failure or myocardial ischemia such as
angina, orthopnea, dyspnea on exertion, exercise intolerance, peripheral edema,
pulmonary rales, S3 gallop, systolic ejection click, or murmur. If she reported
significant symptomatology that was insufficiently evaluated by previous cardiac
workup, I would consider pre-operative echocardiographic evaluation. Cardiac
echocardiography would be helpful in identifying any mitral regurgitation and the
presence or absence of a patent foramen ovale, with the latter being important due to
the increased risk of air embolism when undergoing surgery in the sitting position (in
the presence of a patent foramen ovale, an air embolism may pass into the coronary or
cerebral circulations).

73
Q

Asymptomatic MVP patient getting shoulder surgery:

-After induction the patient is placed in the sitting position. Her blood pressure
drops to 63/38 and heart rate is 90. What do you think is the cause?

WHAT would do?

A

-Induction of anesthesia
-Anaphylaxis
–the development of acute mitral
regurgitation and decreased cardiac output. This may occur because patients
with MVP often experience worsening prolapsed and/or mitral regurgitation with
increased emptying of the left ventricle. (KEEP THAT LEFT VENTRICLE FULL) Therefore, factors such as tachycardia (THEY DON’T WANT TO BE FAST LIKE MR)
(decreased filling time), increased myocardial contractility (sympathetic stimulation
and inotrope administration), decreased systemic vascular resistance (decreased
afterload), hypovolemia (reduced filling), and assumption of the upright posture
(decreased filling) may result in acute mitral regurgitation, decreased cardiac output,
and hypotension.

I would inform the surgeon, return the patient to the supine position,
evaluate the EKG, auscultate the chest for cardiac murmurs and bilateral ventilation,
ventilate with 100% oxygen, give a fluid bolus, administer a pure al-agonist such as
phenylephrine (the tachycardia associated with the administration of an indirect
vasoconstrictor, such as ephedrine, may worsen mitral valve prolapsed and mitral
regurgitation), and consider reducing my anesthetic. During treatment, I would avoid
agents that would increase cardiac contractility and accentuate mitral regurgitation. If
she remained unstable despite these interventions, I would utilize TEE to further
evaluate her cardiac condition.

74
Q

How does anaphylaxis differ from anaphylactoid?

A

Anaphylaxis is a type I hypersensitivity reaction that occurs with the

second exposure to an antigen that previously evoked the production of antigen-
specific IgE antibodies. Degranulation of mast cells and basophils results in the

release of histamine, leukotrienes, prostaglandins, TNF, and various cytokines, with
subsequent increased capillary permeability (histamine, leukotrienes ), peripheral
vasodilation (histamine), bronchoconstriction (histamine, leukotrienes,
prostaglandins ), negative inotropy (leukotrienes ), and coronary artery
vasoconstriction (leukotrienes). The initial manifestations of this life-threatening
reaction usually occur within 10 minutes of exposure to the inciting antigen
The clinical presentation of an anaphylactoid reaction is indistinguishable from
anaphylaxis, with the primary difference being that mast cell and basophil
degranulation is triggered by direct interaction with certain allergens, rather than by
interaction with antigen-specific IgE antibodies. Anaphylactoid reactions, therefore,
do not require prior sensitization and produce anaphylaxis-like symptomatology in a

dose-dependent manner. Classic anaphylaxis, by contrast, does not behave in a dose-
dependent manner, since the immune system is primed to recognize even minute

amounts of the offending allergen and is able to amplify the reaction via IgE
mediation.

75
Q

Assuming she is having a type I hypersensitivity reaction, how would you treat
this condition?

A

In managing this situation, I would inform the surgeon and call for
help; discontinue all infusions and inhalational agents; ventilate with 100% oxygen;
start a 1-2 liter fluid bolus (to replace intravascular volume); infuse intravenous
epinephrine (administer subcutaneously when the patient is normotensive ); administer
corticosteroids (enhances B-agonist effects of other drugs and inhibits the production
of leukotrienes and prostaglandins, but the effects are delayed for 4-6 hours),
histamine blockers, an H2-blocker, and an inhaled beta2 agonist; and provide
supportive care. Recognizing that early intervention with intravenous epinephrine
plays a critical role in reversing the life-threatening events associated with
anaphylaxis, I would double the dose of epinephrine every 1-2 minutes until a
satisfactory systemic blood pressure response was achieved. If her hypotension
proved refractory, I would consider administering bicarbonate to correct any acidemia
( acidemia attenuates the effects of epinephrine on the heart and systemic vasculature ),
starting a vasopressin infusion (often used to treat refractory hypotension associated
with high cardiac output), and evaluating her cardiac function using
echocardiography.

76
Q

How does epinephrine help in the treatment of anaphylaxis?

A

Epinephrine’s u-agonist activity leads to vasoconstriction and reversed
hypotension, while the drug’s B-agonist activity relaxes bronchial smooth muscles
and increases intracellular cAMP, with the increase in intracellular cAMP serving to
restore membrane permeability and decrease the release of vasoactive mediators. The
severity of my patient’s condition would determine the dose and route of
administration. For this patient in complete cardiovascular collapse, I would start
with a 100 μg (range of 100 μg-1 mg) intravenous dose of epinephrine. If, however,
my patient were hypotensive, but not in complete cardiovascular collapse, I would
start with a 10 μg intravenous dose of epinephrine. Finally, if my patient were
normotensive, I would avoid intravenous epinephrine and administer 0.3-0.5 mg
subcutaneously. In all cases, I would double and repeat epinephrine dosing every 1-2
minutes until I achieved an adequate cardiovascular response.

77
Q

How does epinephrine help in the treatment of anaphylaxis?

A

Epinephrine’s u-agonist activity leads to vasoconstriction and reversed
hypotension, while the drug’s B-agonist activity relaxes bronchial smooth muscles
and increases intracellular cAMP, with the increase in intracellular cAMP serving to
restore membrane permeability and decrease the release of vasoactive mediators. The
severity of my patient’s condition would determine the dose and route of
administration. For this patient in complete cardiovascular collapse, I would start
with a 100 μg (range of 100 μg-1 mg) intravenous dose of epinephrine. If, however,
my patient were hypotensive, but not in complete cardiovascular collapse, I would
start with a 10 μg intravenous dose of epinephrine. Finally, if my patient were
normotensive, I would avoid intravenous epinephrine and administer 0.3-0.5 mg
subcutaneously. In all cases, I would double and repeat epinephrine dosing every 1-2
minutes until I achieved an adequate cardiovascular response.

78
Q

What are the risk factors for latex allergy?

A

The risk of latex allergy is highest in children with spina bifida due to
repeated latex exposure associated with their increased health care requirements.
Other risk factors include congenital urinary tract abnormalities, undergoing multiple
surgeries or medical procedures, and working in the health care field or rubber
industry. Finally, patients with an allergy to certain foods containing similar
allergens to those found in latex, such as avocados, bananas, chestnuts, kiwis and
passion fruit, may have antibodies that cross-react with latex.

79
Q

Are there any ways healthcare workers can reduce the risk of latex allergy?

A

The most effective way to prevent health care workers from becoming
“latex sensitive” is to reduce work-related exposure by utilizing non-powdered latex
gloves or latex-free gloves. A healthcare worker who develops a skin rash and/or is
suspected of having a latex allergy, should be referred to an allergist for further
evaluation. If a healthcare worker is diagnosed with latex allergy, then strict
avoidance of latex is critical to preventing a potential anaphylactic reaction.

80
Q

How would you perform an axillary block?

A

I would place the patient in the supine position with the arm abducted
and the elbow flexed at 90 degrees. Next, I would prep and drape the axillary area in
a sterile fashion, use ultrasound to identify the axillary artery and fascial sheath, insert
a 22 gauge short bevel needle into the nerve sheath under ultrasound guidance,
aspirate to ensure extravascular needle placement, and inject 15-20cc oflocal
anesthetic. I would then identify the musculocutaneous nerve within the
coracobrachialis muscle and inject 5cc of local anesthetic next to the nerve. If a
tourniquet were required for the surgery, I would perform a subcutaneous field block
just distal to the axilla to block the medial brachial cutaneous and intercostobrachial
nerves.

81
Q

complications of axillary block

A

While the axillary approach to brachial plexus blocks is associated
with a very low complication rate, there are potential complications, including
intravascular injection, intraneural injection, hematoma, infection, and block failure.

82
Q

Which nerve is most commonly missed in axillary block?

A

The musculocutaneous nerve, which provides motor innervation to the
biceps muscle and sensory innervation to the lateral aspect of the forearm, has already
left the sheath at the level of the axilla and, subsequently, is the most commonly
missed. Adequate blockade of the musculocutaneous nerve is achieved by injecting
5cc of local anesthetic into the coracobrachialis muscle.

83
Q

A 72-year-old female undergoing CABG is about to go on bypass. The patient was
given a standard heparin dose, but the ACT is still low.

1) What do you think is the cause?

A

There are several possibilities, including administration of the wrong
medication, an insufficient dose of heparin, infiltration of the IV, inaccurate ACT
measurement (machine malfunction), and heparin resistance. Heparin resistance can
be due to Antithrombin III deficiency, which may result from inherited defects in
production, excessive loss (e.g., nephrotic syndrome), or excess consumption (e.g.,
sepsis, trauma). If Antithrombin III deficiency was thought to be the problem, then
the treatment would be to administer FFP.

84
Q

Why give FFP for heparin resistance?

A

It has antithrombin 3

85
Q

What steps can you take to prevent a type III protamine reaction?

A
UBP Answer: There is no reliable way to prevent this type of reaction, but diluting
the protamine (e.g., dilute in 50-100 cc and infuse via micro drip) and administering it
slowly (e.g., over at least 5-10 minutes) seems a reasonable approach.
86
Q

Would you infuse protamine via pulmonary artery catheter (PAC) or inject the
medicine directly into the bypass circuit?

A

UBP Answer: I would not administer it directly into the PAC since it could cause
pulmonary HTN, nor would I administer it directly into the bypass circuit because it
may result in clot formation in the bypass machine. In general, the route of
administration, central vs. peripheral, does not seem to make a difference in the
likelihood of adverse reactions. However, there is some evidence that aspirin
administered one week prior to CPB may be beneficial.

87
Q

Late decals are:
Normal FHR
What is FHR variability?

A

Since distinct the decelerations are beginning within 10-30 seconds
after the beginning of each uterine contraction, and resolving within 10-30 seconds
following the end of each contraction, I would conclude that this FHR tracing is
showing late decelerations. Moreover, although the baseline heart rate is normal
(110-160 bpm), fetal heart rate variability, defined as fluctuations in the FHR of more
than 2 cycles per minute, appears to be minimal (<5 bpm).

88
Q

What does normal FHR variability mean? What does it mean when the variability is abnormal?

A

minimal FHR variability, on the other hand, appears to be the most significant
intrapartum sign of fetal compromise. Various factors that may cause or contribute to
decreased FHR variability include fetal hypoxia, fetal sleep state, prematurity, fetal
neurologic abnormalities, fetal tachycardia, betamethasone administration, congenital
anomalies, and/or the administration of central nervous system depressants such as
opioids, barbiturates, magnesium sulfate, and benzodiazepines.

89
Q

If this FHR pattern represented late decelerations how would you treat her?

A

(1) inform the obstetrician, provide supplemental oxygen, ensure adequate
uterine displacement (or even turn the patient on her side), discontinue the
administration of oxytocin (a tocolytic may be considered in the presence of tonic
uterine contractions), begin a fluid bolus, and optimize maternal hemodynamics; (2)
perform a careful history and physical focusing on her airway (exam, previous
anesthetics, etc.), coagulation status (platelet count and trend; signs of a primary
hemostatic defect - easy bruising, bleeding at IV sites, etc.), comorbidities, other
complications associated with pre-eclampsia (cerebral edema - change in mental
status, pulmonary edema, hypovolemia), and prenatal course; (3) discuss the obstetric
plan with the obstetrician; and, assuming there were enough time and no
contraindications to regional anesthesia (i.e. coagulopathy), ( 4) recommend the early
initiation of epidural anesthesia in order to avoid the need for general anesthesia
(especially ifher airway was concerning).

90
Q

What other symptoms can autonomic hyperreflexia cause other than HTN and Bradycardia? DDX if in cysto?

A

AH is also associated
with nasal stuffiness, headache, visual changes, dysrhythmias, nausea, confusion, and
difficulty breathing. Left untreated, sudden and sustained hypertension may lead to cerebral,
retinal, or subarachnoid hemorrhage; loss of consciousness; seizures; myocardial ischemia;
dysrhythmias; and pulmonary edema

DDX: bladder rupture, myocardial ischemia, inadequate anesthesia

91
Q

What is the pathophysiology of autonomic hyperreflexia (AH)? When the lesion is above ___, then what is insufficient?

A

Cutaneous (pain) or visceral (i.e. bladder or rectal distention) stimulus below
the level of spinal cord injury results in a reflex sympathetic discharge. Because the area of
the body below the transaction is neurologically isolated, the sympathetic activity in this area
is not modulated by inhibitory impulses from higher central nervous system centers. This
unopposed sympathetic activity leads to vasoconstriction below the level of the lesion, with
reflex vasodilation above the level of the lesion. When the lesion is above T7, vasodilation
above the lesion is insufficient to prevent systemic hypertension, which then stimulates
carotid sinus receptors, leading to reflex bradycardia.

92
Q

When are pts with AH at risk?

Can pts have AH if lesion is below T7?

A

Patients with transections above T 7 are at risk for autonomic hyperreflexia when spinal cord
reflexes return following spinal shock (usually 1 to 3 weeks post-injury, but may persist for
up to 3 months).

It’s rare, but not impossible

93
Q

Can AH be prevented?

A

While
adequate general and regional anesthesia affectively prevent AH, topical anesthesia has
proven unreliable. When it comes to neuraxial anesthesia, spinal anesthesia is more reliable
than epidural anesthesia for labor, perineal surgery, or bladder surgery, presumably due to
more reliable and complete blockade of sacral roots.

94
Q

How are you handling AH-like, how are you going to treat it?

A

Given this patient’s severe hypertension, I would first
remove the inciting stimulus by asking the surgeon to stop operating and by reducing any
bladder distention. I would then place an arterial line for continuous blood pressure
monitoring; administer a short acting direct vasodilator, such as sodium nitroprusside; and
deepen the anesthetic. Finally, I would cancel the case, monitor the patient closely for
subsequent complications (myocardial ischemia, intracerebral hemorrhage, seizures, etc.),
and allow for complete resolution of the autonomic hyperreflexia.

95
Q

Pt got defasculating dose of roc and then sux and doesn’t have twitches: DDX?

A
Pseudocholinesterase deficiency
Drug error 
Equipment error 
electrolyte abnormalities 
hypothermia
myasthenia graves-associated with those who have thyroiditis (as this pt did)
96
Q

In Myasthenia Gravis, could the pts have reduced sux metabolism if they were receiving anti cholinesterase therapy?

A

Yes, because that therapy may result in reduced plasma cholinesterase

97
Q

If MG pt just got sux, and a defasic dose of roc but didn’t have twitches, what could you give, and what would you need to keep in mind?

After receiving neostigmine, the patient’s respiratory effort improves and she is

extubated. Forty-five minutes later, she complains of weakness and difficulty
breathing. What do you think is the cause?

A

I would provide supportive care
(including post-operative ventilation), consult a neurologist, and consider administering an
anticholinesterase to prolong the action of acetylcholine at the postsynaptic membrane (these
drugs may, themselves, have some limited agonist affect at the acetylcholine receptors),
keeping in mind that overaggressive treatment with an anticholinesterase has the potential to
induce a cholinergic crisis (excessive muscarinic effects of acetylcholine ), which may also
result in weakness.

Given her disease process and recent treatment with an anticholinesterase, her
weakness could be the result of either a myasthenic or a cholinergic crisis. Both conditions
result in muscle weakness, salivation, and sweating. The two conditions may be
differentiated by administering lOmg of edrophonium (Tensilon test), with improved strength
implying myasthenic crisis and increased weakness suggesting a cholinergic crisis.
Additionally, an examination of
the patient for other signs of cholinergic crisis, such as ( 1) constricted pupil size (pupils are
dilated in a myasthenic crisis, secondary to sympathetic activation), (2) weakness and muscle
fasciculations (depolarizing phenomenon occurs at the neuromuscular junction), (3)
bradycardia, ( 4) bronchorrhea (an excessive discharge of watery mucous from the lungs),

98
Q

You get called to optho suite after retrobulbar block-pt lost pulse and consciousness: DDX? what are you going to do?

A

Assuming pt still has pulse, I would…
The timing suggests that local anesthetic injected for the retrobulbar block
has spread through the optic nerve sheath and into the central nervous system, resulting in
general anesthesia and apnea. However, I would also consider other possibilities, including
narcotic or anxiolytic overdose, stroke, and cardiac arrest. Therefore, I would quickly assess
the patient; provide supportive care, including intubation and positive pressure ventilation;
review all the medications that were given; and order tests and lab work as indicated.

99
Q

What are the possible complications of retrobulbar block?

A

Complications from retrobulbar block are uncommon, but include direct
trauma to the optic nerve, retrobulbar hemorrhage, increased intraocular pressure (IOP)
secondary to transient globe compression, globe perforation, stimulation of the oculocardiac
reflex, intravascular or intraneural injection, and apnea secondary to local anesthetic spread
into the CNS.

100
Q

A 2-year-old boy presents to the emergency room with difficulty breathing and wheezing. His
parents report that he began to have trouble breathing an hour ago, shortly after an afternoon
snack. They further report that the child has been afebrile, without any signs of recent illness.

1) What do you think is causing his acute respiratory distress? What will you do?

A

DDX:
While this could simply be an asthmatic attack, the occurrence of the child’s
respiratory distress shortly after a snack would be consistent with other potential etiologies
such as foreign body aspiration and food allergy-induced anaphylaxis.

I would perform a careful history to identify
any known food allergies or previous diagnosis of asthma; similar episodes of respiratory
distress; the type of food ingested during the afternoon snack (peanuts, jellybeans, popcorn,
and hotdogs are the foods most commonly associated with foreign body aspiration); and the
occurrence of coughing or choking during food ingestion (suggestive of foreign body
aspiration). Moreover, assuming the child were stable, I would observe his response to a
bronchodilator, recognizing that a lack of response is more consistent with foreign body
aspiration; examine him for additional signs of anaphylaxis, such as pulmonary edema,
hypotension, urticaria, and pruritus; examine him for other signs of foreign body aspiration
such as stridor, hoarseness, aphonia, retractions, and use of accessory muscles of respiration;
and consider a chest radiograph to help identify the presence and location of a foreign body.
Although most foreign bodies are radiolucent, inspiration/expiration chest x-rays may
indirectly identify the location of an aspirated object by demonstrating hyperinflation (air
trapping in the affected lung), atelectasis, and/or mediastinal shifting toward the unaffected
side.

101
Q

The mother tells you that her son’s snack consisted of applesauce and unroasted
peanuts. Since the child is stable, would you order a chest x-ray to potentially identify
an aspirated foreign body?

A

While a chest x-ray may prove helpful in identifying the presence and
location of a foreign body, I would not delay bronchoscopy in this case where the child may
have aspirated an unroasted peanut. Peanut aspiration is not only associated with the
absorption of water and increasing friability over time, but UNROASTED peanuts may lead to a
peanut oil-induced chemical irritation of the airway with subsequent atelectasis and complete
airway obstruction.

102
Q

Assuming this patient’s respiratory distress is secondary to foreign body aspiration,
how would you induce him?

A

In preparing for induction, I would administer metoclopramide to facilitate
stomach emptying in this patient with a history of recent food ingestion. I would also
consider giving glycopyrrolate to reduce airway secretions and attenuate the reflex
bradycardia often associated with airway instrumentation and bronchoscopy. Next, I would
apply the appropriate monitors and ensure the presence of emergency airway equipment, the
ENT surgeon, and a rigid bronchoscope. Given the potential for distal migration of the
foreign body with assisted ventilation, I would perform an inhalational induction using
oxygen and Sevoflurane and attempt to maintain spontaneous ventilation; recognizing that
this method of induction places him at increased risk for aspiration of gastric contents. After
ensuring an adequate depth of anesthesia to prevent coughing, laryngospasm, or
bronchospasm, I would obtain intravenous access, allow the ENT surgeon to perform direct
laryngoscopy, spray the vocal cords with lidocaine (to reduce the risk of laryngospasm
during endoscopy), and insert the rigid bronchoscope. Once the bronchoscope was in place, I
would connect the anesthesia circuit to the bronchoscope to allow for ventilation during
foreign body removal.

103
Q

foreign body aspiration: Shouldn’t you perform a rapid sequence induction, since this patient had a snack just a
couple of hours ago?

A

Despite the risk of aspiration, I would not perform a rapid sequence induction
in this situation, recognizing that positive-pressure ventilation may place the patient at risk
for: (1) distal migration of the foreign body, making extraction more difficult; and (2)
hyperinflation and/or pneumothorax, if the foreign body is producing a ball-valve affect.
Rather, I would perform an inhalational induction with the goal of maintaining spontaneous
respirations. However, I do recognize that there is no evidence that outcomes for this
procedure are altered by the chosen method of ventilatory management during induction (i.e.
spontaneous vs. controlled ventilation), and would consider altering or modifying my
strategy depending on the circumstances (e.g. patient is vomiting preoperatively, further
increasing the risk of aspiration).

104
Q

During attempted removal the foreign body becomes lodged at the carina causing
complete airway obstruction. What would you do?

A

I would immediately inform the surgeon that ventilation has become
impossible and ask him to either quickly remove the foreign body or attempt to push it
distally into one of the main bronchi so that one-lung ventilation was possible. If these
interventions were unsuccessful, I would attempt to improve ventilation by moving the
patient into the lateral or prone position, or by advancing an ETT beyond the obstruction. If
these interventions were also unsuccessful and life-threatening hypoxia was imminent, I
would consider placing the patient on cardiopulmonary bypass until the obstruction could be
cleared.

105
Q

What are the complications that may occur with foreign body aspiration?

A

Complications arising from foreign body aspiration depend on the location
and type of foreign body aspirated (organic vs. non-organic, sharp vs. dull), and the duration
of time the foreign body remained in the airways. Nuts and certain vegetable materials are
highly irritating to the bronchial tree and produce a chemical pneumonitis. If the foreign
body is successfully removed within 24 hours of the incident, the complication rate is very
low. However, the longer the foreign body remains in the airways, the more likely
complications such as bronchial stenosis, bronchiectasis, pneumonia, lung abscess, tissue
erosion/perforation, and pneumomediastinum or pneumothorax will occur.

106
Q

What’s the concern with giving an epidural to a pt who has HOCM?

A

The concern in providing neuraxial
anesthesia to a patient with hypertrophic obstructive cardiomyopathy (HOCM) is that a
sympathectomy-induced reduction in systemic vascular resistance (SVR) may lead to
reduced preload and the subsequent development or exacerbation of left ventricular outflow
tract (L VOT) obstruction. However, as long as euvolemia is maintained, epidural anesthesia
may be safely employed to achieve levels sufficient for vaginal or surgical delivery.

107
Q

What is HOCM? Definitive diagnosis?

A

Hypertrophic obstructive cardiomyopathy (HOCM) is a genetic condition
characterized by left ventricular hypertrophy, systolic anterior movement of the mitral valve
(SAM), dynamic left ventricular outflow tract (L VOT) obstruction, decreased left ventricular
size, diastolic dysfunction, myocardial ischemia (even in the absence of coronary artery
disease), and dysrhythmias (these dysrhythmias are responsible for the sudden death that may
occur in young adults with HOCM). In patients with HOCM, hyperdynamic left ventricular
contraction of the hypertrophied septum results in the rapid movement of blood through the
narrowed L VOT creating a Venturi effect on the anterior leaflet of the mitral valve. This
Venturi effect leads to SAM, which in turn leads to dynamic LVOT obstruction and mitral
regurgitation.

Diagnosis: Echo
Definitive diagnosis:endomyocardial biopsy and DNA analysis

108
Q

What makes HOCM worse?

A

The dynamic outflow obstruction associated with HOCM is accentuated by
any intervention or event that results in a reduction in left ventricular end diastolic volume.
Therefore, the obstruction is potentially exacerbated by the following conditions: (1)
hypovolemia (reduced preload); (2) sympathectomy and/or vasodilation (decreased
SVR/afterload, which facilitates left ventricular emptying); (3) increased myocardial
contractility (increased left ventricular emptying); (4) tachycardia (reduced diastolic filling
time); (5) sympathetic stimulation (tachycardia and/or increased inotropy); (6) dysrhythmia
(inadequate ventricular filling); (7) excessive positive-pressure ventilation and/or PEEP
(decreased preload); and, in this case, (8) inadequate left uterine displacement (decreased
preload).

109
Q

HOCM pt has boggy uterus-what are you going to give/do?

A

Recognizing that the systemic vasodilation and reflex tachycardia often
associated with the administration of oxytocin could accentuate L VOT obstruction, I would
consider administering an intramuscular injection of methylergonovine (methergine) or
intravenous 15-methyl prostaglandin F2a (Hemabate) to induce uterine contraction. I would
avoid the intravenous injection of methergine due to the increased risk of acute hypertension,
seizures, cerebrovascular accident, retinal detachment, and myocardial arrest associated with
this route of administration. If I believed it was necessary to administer oxytocin, I would
give it very slowly to reduce the risk of a significant reduction in SVR.

110
Q

HOCM pt: Shortly after delivery, the patient develops dyspnea and pulmonary edema. What is
your differential?

A

The diastolic dysfunction associated with HOCM places these patients at
increased risk of developing pulmonary edema with fluid overload. Therefore, the timing of
this event suggests that her pulmonary edema may be secondary to the abrupt autotransfusion
of blood into the central circulation that occurs with post-delivery uterine contraction and
involution. However, recognizing that patients with HOCM are at increased risk of
developing myocardial ischemia, I would also consider the possibility of left heart failure
secondary to myocardial ischemia or infarction. Another consideration would be that the
“bearing down” (valsalva-induced reduction in preload) and/or pain (sympathetic-induced
tachycardia and/or inotropy) occurring during delivery resulted in worsening LVOT
obstruction, with subsequent left heart failure. Other considerations would include
hypovolemia (excessive blood loss secondary to uterine atony or other cause), a
sympathectomy-induced decrease in SVR (due to excessive level of neuraxial anesthesia),
cardiac arrhythmia, thrombotic embolism (pregnant patients are hypercoagulable ), and
amniotic fluid embolism.

111
Q

HOCM pregnant pt who just delivered: Assuming her pulmonary edema was secondary to worsening LVOT obstruction, how
would you treat her condition? If intubation becomes necessary, how would you manage vent settings?

A

My treatment would be aimed at the correction or elimination of factors that
could be accentuating her dynamic L VOT obstruction. Therefore, I would ensure adequate
volume replacement and treat any hypotension, tachycardia, and/or dysrhythmia. More
specifically, I would administer a ~-blocker to slow her heart rate, prolong diastolic filling
time, and decrease myocardial contractility. If she were hypotensive, I would administer
phenylephrine (the tachycardia associated with ephedrine administration would be
undesirable) and ensure adequate volume replacement. If intubation and positive pressure
ventilation became necessary, I would perform a rapid sequence induction (assuming a
reassuring airway) and begin positive pressure ventilation with smaller tidal volumes and an
appropriately increased ventilatory rate (to avoid the decreased preload associated with
excessive intrathoracic pressures). While PEEP and diuretics are often utilized in the
treatment of pulmonary edema, these interventions would potentially reduce her cardiac
preload, thereby accentuating LVOT obstruction, the primary cause of her pulmonary edema.

112
Q

If you had been the intensive care physician responsible for this patient with
status asthmaticus four days ago, how would you have treated her condition?

A

When treating a patient with this life-threatening condition, I would:
(1) provide supplemental oxygen to maintain the oxygen saturation above 90%; (2)
administer B2-agonists (i.e. albuterol), corticosteroids (recognizing that it may take 4-
6 hours to realize a therapeutic benefit), aminophylline (to induce bronchodilation,
stimulate the central respiratory cycle, reduce diaphragmatic muscle fatigue, and relax
vascular smooth muscles), empirical broad-spectrum antibiotics, and intravenous
fluids (although the benefits of this treatment are limited); (3) order pulmonary
function tests and arterial blood gasses to monitor the adequacy of oxygenation,
ventilation, and the patient’s response to treatment; (4) consider the addition of
intravenous magnesium sulfate (for bronchodilatory affects), if the patient’s response
to other bronchodilators is inadequate; and (5) consider mechanical ventilation, if the
patient begins to show signs of respiratory fatigue and/or inadequate ventilation and
oxygenation (PaC02 > 50 mmHg).

113
Q

When would you intubate a patient with status asthmaticus, and what ventilator
strategy would you employ? And when would you begin weaning?

A

I would consider intubation and the initiation of mechanical
ventilation ifthe patient began to show signs of respiratory fatigue and/or inadequate
ventilation and oxygenation. Therefore, I would monitor the patient’s response to
therapy using pulmonary function tests and arterial blood gasses. If pulmonary
function testing showed a FEV1 or peak expiratory flow rate :S 25% of normal, or if
arterial blood gasses showed a PaC02 > 50 mrnHg, despite aggressive therapy, I
would intubate the patient and initiate mechanical ventilation.
Therefore, I would employ a pressure control mode of
ventilation, recognizing that the decelerating flow pattern associated with this mode
of ventilation will more efficiently overcome the high resistance of the asthmatic’s
airways, minimize the peak pressures required to deliver a given tidal volume, andimprove the distribution of ventilation. Moreover, I would establish a prolonged
expiratory phase to allow for complete exhalation and to avoid auto-PEEP (breath
stacking), which can result in barotrauma. When the patient’s FEV1 or peak
expiratory flow rates increased to 2: 50% of normal, I would initiate weaning from
mechanical ventilation.

114
Q

Ok. Back to our current situation, with the patient having been discharged from
the hospital several days ago and now presenting to the operating suite with
acute appendicitis. How would you assess this patient’s asthmatic condition,
pre-operatively?

A

I would first perform a careful history, focusing on the severity and
characteristics of her pulmonary disease, along with the effectiveness of her current
therapy. To this end, I would attempt to elicit information concerning the age of
onset, triggering events, allergies, recent respiratory infection, changes in
symptomatology (cough, sputum, wheezing, etc.), current medications, anesthetic
history, and her recent hospital course. Next, I would perform a physical exam to
identify any pulmonary wheezing or crepitations and/or the use of accessory muscle
of respiration. Considering the severity of her disease, I would order: (1) pulmonary
function tests, before and after bronchodilator therapy, to more accurately assess the
severity of obstruction and her response to therapy; (2) arterial blood gasses, to
evaluate the adequacy of ventilation/oxygenation and to establish baseline levels
(helpful in the event of subsequent respiratory dysfunction); and (3) chest x-rays, to
identify or rule out pulmonary infection.

115
Q

How would you prepare her for emergent surgery?

A

My goals in preparing this patient for surgery are to optimize her
asthma, control her pain, reduce her anxiety, and minimize the risk of aspiration.
Therefore, I would reassure the patient and family, continue her current medications,
and consider chest physiotherapy. Moreover, I would administer fentanyl, to avoid
the pulmonary splinting, decreased ability to cough, and bronchospasm potentially
associated with pain (avoid narcotics that release histamine and carefully titrate to
avoid respiratory depression); diphenhydramine (an H1-receptor blocker), to inhibit
histamine-induced bronchoconstriction and reduce the potential for anxiety-induced
bronchospasm; a stress dose of hydrocortisone (100 mg), given the potential for
hypothalamic-pituitary-adrenal suppression with chronic steroid treatment (she is
being treated with dexamethasone ); metoclopramide, to facilitate stomach emptying;
and ondansetron, to treat her nausea. Just prior to induction, I would administer a
short acting B2-agonist (i.e. albuterol), to minimize the risk of bronchoconstriction
during intubation.

116
Q

Would you give atropine, pre-operatively?

A

Anticholinergic medications may be beneficial for asthmatic patients
secondary to reduced mucous gland secretions (possibly improving inflammation)
and airway hyperreactivity (secondary to reduced vagal tone and inhibition of
muscarinic cholinergic receptors). However, their preoperative administration is
controversial, since they could result in increased inspissation (increased viscosity
and thickening of airway secretions), potentially leading to airway plugging and the
initiation of an asthmatic attack. Therefore, considering these potential
complications, and recognizing that the intramuscular doses of anticholinergic
medications typically used for pre-anesthetic medication are unlikely to significantly
decrease her airway resistance (they would be sufficient to reduce airway secretions),
I would not administer this medication pre-operatively. If I wanted to administer an
anticholinergic, preoperatively, to optimize his asthmatic condition, I would consider
providing an inhaled medication, such as ipratropium.

117
Q

You are planning general anesthesia for the procedure. How will you induce
her?

A

My goals when inducing this severely asthmatic patient with a full
stomach and nausea, are to achieve an adequate plane of anesthesia to avoid
bronchoconstriction in response to mechanical stimulation while, at the same time,
minimizing the risk of aspiration. Therefore, assuming her airway exam were
reassuring, I would: (1) administer a short acting B2-agonist; (2) denitrogenate with
100% oxygen; (3) ensure that she had received metoclopramide and ondansetron to
facilitate gastric emptying and treat her nausea, respectively; (4) administer 2μ/kg of
fentanyl, 2-3 minutes prior to induction, to avoid light anesthesia during
laryngoscopy; (5) give 1-2 mg/kg of intravenous lidocaine, 1-2 minutes prior to
induction, to prevent reflex-induced bronchoconstriction (Topical lidocaine may also
be used, but the application may provoke bronchospasm if the depth of anesthesia is
insufficient. Since there is significant risk for light anesthesia during a RSI, I would
not employ this technique.); (6) apply cricoid pressure; and (7) perform a RSI using
ketamine (induces bronchodilation), propofol (produces bronchodilation and a more
reliable depth of anesthesia as compared to thiopental), and succinylcholine. While
succinylcholine could potentially result in significant histamine release (risk for
histamine-induced bronchospasm), I believe that its ability to facilitate the rapid
placement of an endotracheal tube is important to reduce the risk of aspiration in this
patient presenting for emergent surgery (inadequate fasting), an acute abdominal
process (delayed gastric emptying), and active nausea.

118
Q

Pt just got a block and had a seizure:

A

While I would consider several potential causes of these symptoms,
such as hypoxia, acidosis, myocardial ischemia, alcohol withdrawal, and a seizure
disorder, the timing of the event and the progression of his symptomatology are most
consistent with local anesthetic toxicity. Recognizing that the presentation of local
anesthetic toxicity is extremely variable in onset and initial symptomatology, I would
consider this possibility in any situation where a patient experienced an altered mental
state, neurologic symptoms, or cardiovascular instability following the administration
of local anesthetic for regional anesthesia.

119
Q

What are the signs and symptoms associated with local anesthetic toxicity?

Do patients always have these symptoms? Can LAST be delayed?

A

Initially, the patient developing local anesthetic toxicity may
experience nonspecific neurologic symptoms such as metallic taste, circumoral
paresthesias, tongue numbness, visual disturbances (i.e. blurred vision and difficulty
focusin), auditory disturbances (i.e. tinnitus), lightheadedness, dizziness, and a feeling
of “impending doom”.

When administering local anesthetics, it is important to keep in mind that patients
may progress rapidly to seizure activity and cardiac toxicity without experiencing any
of the initial nonspecific neurologic symptoms, as may occur following a direct
arterial injection. Likewise, the onset of symptoms may be significantly delayed, as
may be the case following tumescent procedures (delayed for over 15 minutes in
some reports).

120
Q

Are there any advantages to using ropivacaine rather than bupivacaine?

A

In the case of ropivacaine, the reduced
cardiac depression of its propyl side chain, as compared to the butyl side chain of
bupivacaine, may also play a role. Another potential advantage of ropivacaine is the
provision of similar sensory blockade in association with less extensive motor
blockade, as compared to an equal dose ofbupivacaine.

121
Q

Does the addition of epinephrine reduce the risk of local anesthetic toxicity?

A

The addition of epinephrine may reduce the risk of local anesthetic
toxicity by reducing systemic absorption (secondary to vasoconstriction) and/or
helping to identify unintended intravascular injection (i.e. an increase in heart rate of
IO beats/minute or an increase in systolic pressure of> 15 mmHg).

122
Q

How do local anesthetics affect the heart?

A

The inhibition of voltage-gated sodium channels results in the
following direct cardiac effects: (1) slowed cardiac conduction (increased PR-interval
and widened QRS complex), (2) decreased rate of depolarization (secondary to a
reduction in availability of the fast sodium channels that allow the rapid sodium
influx required for membrane depolarization), (3) a dose-dependent reduction in
cardiac contractility (potentially contributing to hypotension, metabolic acidosis, and
reduced clearance oflocal anesthetic), and (4) depressed spontaneous pacemaker
activity in the sinus node (potentially leading to bradycardia and cardiac arrest).
Peripheral vascular effects vary depending on the amount of local anesthetics in the
plasma, with low concentrations resulting in (5) vasoconstriction and high
concentrations causing (6) vasodilation.

123
Q

The patient’s blood pressure is stable, but he continues to experience seizure
activity and develops stable monomorphic ventricular tachycardia. Assuming
his condition is the result of local anesthetic toxicity, what will you do?

A

In treating this complication, I would: (1) stop injecting local
anesthetic; (2) call for help and a lipid rescue kit; (3) ensure adequate ventilation and
oxygenation to correct and/or avoid factors that enhance the systemic toxicity of local
anesthetics, such as hypercarbia (increased cerebral blood flow, intra-neuronal ion
trapping of the drug, and decreased plasma protein binding of local anesthetics),
acidosis (reduces the seizure threshold and decreases plasma protein binding of local
anesthetics), and hypoxemia; (4) administer a benzodiazepine to treat his seizure
(seizure activity increases metabolism, which may lead to hypoxemia, hypercarbia,
and acidosis); (5) administer succinylcholine and intubate the patient if ventilation
were inadequate, the risk of aspiration was significant (history of GERD or hiatal
hernia), or if tonic-clonic movements persisted despite benzodiazepine administration
(while succinylcholine would minimize the metabolic acidosis associated with
seizure-induced muscle activity, it would not affect the acidosis that develops
secondary to seizure-induced increases in cerebral metabolism); (6) initiate lipid
emulsion therapy with a bolus of 1.5 mL/kg of 20% lipid solution (roughly 100 mL in
adults) and a continuous infusion at 0.25 mL/kg/minute, with plans to discontinue the
infusion only after establishing hemodynamic stability for at least 10 minutes (repeat
bolus and double infusion rate every 5 minutes as necessary, keeping in mind that the
recommended upper limit for initial dosing is 10 mL/kg for 30 minutes); (7)
administer adenosine and/or amiodarone for additional treatment of his ventricular
dysrhythmia (procainamide, lidocaine, ~-blockers, calcium channel blockers, and
vasopressin should be avoided when treating bupivacaine-induced ventricular
arrhythmias); (8) perform immediate synchronized cardioversion if the patient
became unstable (assuming he remained in monomorphic VT- polymorphic VT
usually requires unsynchronized shock); and (9) consider cardiopulmonary bypass if
the patient’s response to these therapies was inadequate (cardiopulmonary bypass
may serve as “bridging therapy” until tissue levels of local anesthetic have
diminished).

124
Q

epi, vaso, calcium channel blockers, local anesthetics, and beta blockers in LAST:

A

Epinephrine - Standard resuscitation doses of epinephrine (1 mg) are not
recommended during resuscitation of a patient experiencing local
anesthetic toxicity because epinephrine is highly arrhythmogenic and can
reduce the efficacy of lipid rescue. Therefore, it is recommended utilize
smaller doses in this setting (<1 μg/kg or 10-100 μg boluses).

No vaso
No beta blockers
No calcium channel blockers
No lidocaine or procainaide (local anesthetics)

125
Q

Propofol in last while the Nurse runs to get midazolam?

A

Given the potential for cardiovascular instability in this situation, I
would not administer propofol. While propofol and thiopental are acceptable
alternatives to quickly stop seizure activity, they should be avoided in the setting of
cardiovascular instability because of their direct cardiodepressant effects.
Furthermore, the low lipid content of propofol makes it an inappropriate substitute for
lipid emulsion therapy.

126
Q

When is the appropriate time to initiate lipid emulsion therapy?

A

The appropriate time to initiate lipid emulsion therapy is controversial.
Since early treatment may prevent cardiovascular collapse, many practitioners believe. that waiting until standard therapy has failed to initiate lipid emulsion therapy is
unreasonable. On the other hand, administering lipids at the first sign of local
anesthetic toxicity would result in unnecessary treatment, since only a fraction of
patients progress from the initial premonitory symptoms to severe toxicity.
Therefore, I would initiate therapy when the signs and symptoms of local anesthetic
toxicity appeared to be rapidly progressing or when a patient experienced prolonged
seizure activity or signs of cardiac toxicity (i.e. bradycardia, heart block, hypotension,
asystole, or ventricular arrhythmia).

127
Q

How long do you watch a patient that has LAST?

A

Patients should be monitored for at least 30 minutes following the administration
of potentially toxic doses of local anesthetic because toxicity has been reported to
present longer than 15 minutes after injection (this recommendation refers to
patients who have yet to develop any signs or symptoms of toxicity).
• Patients should be monitored very closely for at least 12 hours following
significant local anesthetic toxicity because local anesthetic can continue to
redistribute to the circulation from tissue depots, potentially resulting in delayed
recurrence of severe toxicity.

128
Q

How does shock wave lithotripsy disintegrate renal calculi without causing an
unacceptable amount of tissue damage?

A

The acoustic impedance of water and body tissues is similar, the
shock wave travels through body tissues without a significant dissipation of energy,
causing minimal tissue damage.

129
Q

Anesthesia for lithotripsy:

What if you were concerned about airway?

A

While intravenous analgesia and sedation are usually adequate for
procedures performed with second and third generation lithotripters, the increased
discomfort associated with the more powerful first-generation lithotripters often
requires general anesthesia, neuraxial anesthesia, or flank infiltration with/without
intercostal blocks.

If I was concerned about airway, I could do a spinal or epidural of course after looking at CBC and coags and assuming the pt not on AC

130
Q

Lithotripsy: first generation machine:Could you proceed with flank infiltration and intercostal nerve block
placement?

A

While this technique is a viable option for the procedure, I believe that
neuraxial anesthesia would more reliably provide adequate analgesia (as compared to
flank infiltration combined with intercostal nerve blocks), thus reducing the potential
necessity for additional sedation. Since one of my principal goals in managing this
patient with a potentially difficult airway is to maintain spontaneous ventilation and
avoid the necessity for airway manipulation, I would prefer to proceed with the
technique that most reliably avoids the need for supplemental intravenous sedation.

131
Q

What are the risks associated with performing ESWL on a patient with an
AICD?

A

While the overall risk is low, there is some risk that patients with a
cardiac rhythm management device (CRMD), such as a pacemaker or automatic
implantable cardioverter-defibrillator (AICD), will experience shock wave-induced
intraoperative arrhythmias during ESWL (as are those with a history of arrhythmias).
Moreover, lithotripter-induced shock waves can lead to CRMD malfunction, such as
switching to magnet mode, pacing suppression, oversensing of asynchronous shocks,However, as long as the CRMD generator is not located in the abdomen (usually
located in the pectoral region), ESWL is not contraindicated in patients with these
devices.

132
Q

How are you managing AICDs in pts getting lithotripsy:

A

(1) ascertain the
indication for placement, the patient’s underlying rhythm and rate, and the degree of
pacemaker dependency; (2) determine the type, manufacturer, programmability, and
functionality of the device (i.e. battery life, lead integrity, the presence of any alert
status, and sensing/pacing thresholds); (3) verify the behavior of the device when
exposed to a magnet (usually disables tachydysrhythmia detection and therapy); (4)
ensure the availability of a programming device, trained pacemaker programmer, and
alternative pacing modality in the operating room; (5) make sure that the patient’s
CRMD is not in the shock wave path (the focal point of the lithotripter should be kept
at least six inches away from the pacemaker); (6) employ continuous telemetry; (7)
begin lithotripsy with low energy shock waves followed by gradually increasing
energy levels, while closely monitoring pacemaker function; (8) terminate lithotripsy
if the patient developed an arrhythmia; and (9) use a magnet only if there were
inhibition of the device’s pacemaker function.
I would not require preoperative interrogation as long as his device had been checked
within the last 6 months (Recommendation: implantable cardioverter-defibrillators
should ideally be checked within last 6 months, and pacemakers within the last 12
months).

133
Q
Post op (after lithotripsy): Postoperatively his hemoglobin has dropped from 14 mg/dL to 10.4 mg/dL. Are
you concerned? What do you think is the cause?
A

Tell surgeon!!! Always communicate with them!!!!!
A drop of 3.6 mg/dL in the hemoglobin concentration is significant,
and would elevate my suspicion of intra-abdominal or retroperitoneal hemorrhage. In
evaluating his progressive anemia, I would examine the patient’s abdomen, stabilize
his hemodynamics, look for other sources of bleeding, and consider radiographs or
CT to identify any hematoma formation. During this evaluation, I would also
consider other potential causes of his postoperative anemia, such as hemodilution
from excessive fluid administration and/or shock wave-induced damage to the
gastrointestinal, pulmonary, or urinary systems.

134
Q

A 68-year-old male, presents for total laryngectomy to remove a glottic mass after failed
radiation therapy. He is an alcoholic who has been smoking cigarettes for over 40 years.
How would you evaluate this patient airway preoperatively?

A

Given the potential for difficult intubation and ventilation, I would: ( 1)
perform a standard airway exam, including mouth opening, neck range of motion,
thyromental distance, etc.; (2) review the surgeons notes from the most recent fiberoptic and
mirror inspection, to help identify the extent of the mass; (3) look at the most recent CT scan
of his neck to identify and assess the extent of airway compromise; ( 4) palpate the patient’s
neck to identify any masses and/or tracheal deviation, and to assess tissue plasticity (the latter
may be affected by mass affect and/or radiation therapy); and (5) further examine the patient
for signs of airway obstruction, such as stridor, hoarseness, dyspnea, sternal retraction, and
the use of accessory muscles of respiration.

135
Q

Glottic mass/total laryngectomy pt: On exam, you note stridor, sternal retraction, and the use of accessory muscles of
respiration. Would you perform an awake fiberoptic intubation?

A

While an awake fiberoptic intubation is often utilized in the management of a
known or anticipated difficult airway, it may not be the best approach in the setting of
advanced obstructive laryngeal disease due to: (1) the risk of complete obstruction as the
fiberscope passes through the mass; (2) the difficulty of achieving adequate airway analgesia
in the setting of advanced laryngeal pathology; and (3) the technical difficulty of performing
fiberoptic intubation through a large vascular friable tumor (i.e. tortuous route, bleeding,
and/or edema). Therefore, I would consider one of the following options: (1) an inhalational
induction with the goal of maintaining spontaneous ventilation until the airway was secured;
(2) placement of a transtracheal catheter under local anesthesia, followed by intravenous
induction and jet ventilation through the catheter; or (3) an awake elective tracheostomy
under local anesthesia.

136
Q

Total laryngectomy pt:
Later that night you are called to the ICU because the patient’s tracheostomy tube was
accidentally removed. Your attempts to reinsert the tracheostomy tube are
unsuccessful. What would you do?

A

Recognizing that there is no longer a communication between the oropharynx
and the lower trachea in a patient who has undergone total laryngectomy, I would have
someone call for a surgeon capable of obtaining emergency airway access. In the meantime,
I would attempt to oxygenate and ventilate the patient via a partially inflated LMA applied
over the stoma site, or jet ventilation through a catheter inserted into the stoma.

137
Q

You provide anesthesia for a 2-year-old female for bilateral myringotomy tube placement.
You are checking on the child in the postoperative unit, when her heart rate suddenly
speeds up to 190 bpm. What would you do?
Dose of adenosine in kids ?

A

Assuming she were hemodynamically stable and that I believed this to be

supraventricular tachycardia (i.e. heart rate > 180 bpm in a child, absent or abnormal P-
waves, and/or a history of abrupt rate changes), I would first apply supplemental oxygen and

attempt to convert her arrhythmia with a vagal maneuver, such as applying ice to her face. If
this were ineffective, I would establish intravenous access and administer 0.1 mg/kg of
adenosine in an attempt to interrupt any reentry circuit involving the atrioventricular node
(the usual cause of SVT in children) by temporarily blocking conduction through the
atrioventricular node. If her SVT persisted or reoccurred, I would give successive doses of
0.2 and 0.4 mg/kg every 1-2 minutes, as necessary. At the same time, I would ensure
adequate monitoring (i.e. ECG, blood pressure cuff, pulse oximeter), secure her airway
(preferably an ETT), prepare for cardioversion in case the patient became unstable, and treat
any possible contributing factors, especially hypoxia.

138
Q

2 yo with HR of 190 (cont)
The vagal maneuver is unsuccessful and you do not have intravenous access. What
would you do?

A

Assuming she remained hemodynamically stable, I would make a quick
attempt at obtaining peripheral intravenous access. If I were unable to quickly achieve this, I
would immediately insert an interosseous needle into the anterior surface of her tibia (1-2 cm
below and 1 cm medial to the tibial tuberosity). While an interosseous line provides good
access for the administration of all fluids and medications routinely given duringcardiopulmonary resuscitation, it is also associated with several complications, such as
osteomyelitis, fat and bone marrow embolism, and compartment syndrome. Therefore, I
would view this as a temporary solution until a more definitive intravenous line could be
placed.

139
Q

1 yo with HR of 190 (cont):

Would you give adenosine through a peripheral line?

A

While central line access may be preferable due to the rapid metabolism of
adenosine by red blood cell adenosine deaminase (half-life= 10 seconds), I would administer
it through a peripheral line if necessary. However, given its rapid metabolism, I would
quickly flush the peripheral line with 10 mL of saline following drug administration.

140
Q

2 yo with HR of 190 cont:
Before you have a chance to establish intravenous access of any kind, her heart rate
increases to 260 bpm and her blood pressure drops precipitously. What would you do?

A

In the setting of supraventricular tachycardia and hemodynamic instability, I
would prepare for immediate synchronized cardioversion, beginning at 0.5 J/kg and doubling
the energy dose as required up to 2 J/kg. While cardioversion was being prepared, I would
secure her airway, apply 100% oxygen, and attempt to place an interosseous line (in the
setting ofhemodynamic instability, time should not be wasted attempting to obtain
intravenous access, which may prove more difficult secondary to poor peripheral perfusion).
However, given the urgency of converting her arrhythmia, I would not delay cardioversion
for intravenous access.

141
Q

PCN allergy and cephalosporin: You giving it? What if pt had difficulty breathing?

A

While I understand that the true cross-reactivity between penicillin and
cephalosporins is about 0.5% for first-generation cephalosporins and near zero for both
second and third-generation cephalosporins, I would not administer a cephalosporin to a
patient who had experienced anaphylaxis following penicillin administration due to the
potentially serious consequences of this type ofreaction. Therefore, since this patient’s
history of difficulty breathing is consistent with an anaphylactic response, I would suggest
utilizing vancomycin rather than a cephalosporin for antibiotic prophylaxis.

142
Q

You decide to order vancomycin for antibiotic prophylaxis. The nurse calls to tell you
that the patient is “very red” and his blood pressure has dropped. What do you think is
going on? What could you do to prevent this?

A

The timing of his hypotension, along with flushing (“very red”) and upper
body erythema, is consistent with “red man’s syndrome”, which results when rapid
administration of vancomycin leads to histamine release. While the most common
manifestation of rapid administration ( 10-15 mg/kg should be administered over 60 minutes)
is isolated hypotension, other symptoms such as pruritus, flushing, upper body erythema, and
even cardiac arrest may occur. Giving antihistamines, such as diphenhydramine (H1-receptor
antagonist) and cimetidine (H2-receptor antagonist), one hour before vancomycin
administration may attenuate the drop in systemic vascular resistance associated with
histamine release.

143
Q

What are the causes of arachnoiditis? How is it diagnosed?

A

There are several potential causes of arachnoiditis (inflammation of the
meninges and subarachnoid space), such as: (I) direct injury to the spine, as may occur with
trauma, spinal surgery, and multiple lumbar punctures (especially when bleeding into the
spinal fluid occurs); (2) infections, such as tuberculosis or viral and fungal meningitis; (3)
chemicals, such as contrast dye, disinfectants, and preservatives found in local anesthetic and
epidural steroid preparations; and ( 4) chronic compression of spinal nerves, as may occur
with degenerative disc disease and/or severe spinal stenosis.

Symptoms associated with arachnoiditis include: (1) back pain that increases
with activity; (2) various sensory and motor abnormalities, such as tingling, numbness,
weakness, cramps, spasms, and severe shooting leg pain; and (3) bowel, bladder, and/or
sexual dysfunction. Computerized axial tomography and magnetic resonance imaging are
often used to help establish the diagnosis.

144
Q

Treatment of arachnoiditis:

A

Since there is no cure for arachnoiditis, the treatment is similar to that utilized
for other chronic pain conditions. Treatment modalities include physical therapy, pain
medications (i.e. NSAIDs, narcotics, corticosteroids, anti-spasm drugs, anti-convulsants (help
with burning pain), intrathecal pump placement, transcutaneous electrical nerve stimulation,
and/or a spinal cord stimulator.

145
Q

A 52-year-old male presents for debridement and repair of injuries to his right elbow and
forearm sustained in a bar fight a couple hours earlier. He admits to a 34-year smoking
history.
The surgeon requests that the procedure proceed with an interscalene block. Would
you agree?

A

Assuming this patient who was recently in a bar fight would tolerate
undergoing the procedure under regional anesthesia (i.e. not intoxicated or combative), and
that his long-term tobacco use had not resulted in significant pulmonary disease (may not
tolerate hemidiaphragmatic paralysis), I would consider an interscalene block for this
procedure. However, since the surgical procedure involves the forearm, an axillary block or
supraclavicular block may be more appropriate considering the ulnar nerve sparing that is
often associated with an interscalene block (the C8-Tl nerve roots, which form the ulnar
nerve, are spared with an interscalene block up to 50% of the time). Therefore, ifthe surgeon
and patient preferred an interscalene block for this procedure, I would let them know that the
ulnar nerve may need to be blocked with a separate injection.

146
Q

The surgeon wants to use a tourniquet. What are the complications are associated with
tourniquet use?

A

There are a number of complications associated with prolonged tourniquet
use, dull achy pain, tachycardia, and hypertension; (2) muscle damage, occurring beneath the
tourniquet at 2 hours (direct pressure), and distal to the tourniquet at 4 hours (muscular
ischemia, edema, and microvascular congestion); (3) nerve injury, occurring secondary to
direct pressure under the cuff (some sources say ischemia contributes) ; ( 4) thrombotic
embolism, occurring with tourniquet release; (5) vascular injury, most commonly in patients
with peripheral vascular disease; (6) undesirable drug effects (inadequate antibiotic tissue
concentrations when given after tourniquet inflation; tissue sequestration of drugs
administered prior to inflation, followed by release and subsequent affect following deflation;
and reduced volume of distribution for drugs administered following inflation); (7) transient
metabolic acidosis, occurring with deflation and the washout of accumulated metabolic waste
products in the ischemic extremity; and (8) skin damage, such as pressure necrosis, friction
bums, and chemical bums (the latter resulting from antiseptic skin preparation solutions
becoming trapped under the cuff and compressed against the skin).

147
Q

An hour into the procedure, the patient begins to complain of pain at the site of the
tourniquet despite adequate surgical analgesia. What do you think? And what nerves (other than ulnar) are spared with an interscalene block?

What would you do?

A

While the exact mechanism of tourniquet pain is unknown, it is believed that
pain transmission through unmyelinated C-fibers occurs with the recession oflocal anesthetic
blockade. The pain, which occurs despite adequate surgical analgesia, is often described as a
dull ache or burning sensation, begins around 45-60 minutes following tourniquet inflation,
and is associated with tachycardia and hypertension. This patient, however, may be
experiencing pain secondary to inadequate analgesia of the medial brachial cutaneous and
intercostobrachial nerves supplying cutaneous innervation to the medial aspect of the upper
arm, since these nerves are spared with an interscalene block.

Recognizing that the definitive treatment for tourniquet pain is to release the
tourniquet, I would ask the surgeon to allow deflation as soon as possible. In the meantime, I
would provide small doses of an opioid and midazolam to ease his discomfort, with the goal
of maintain adequate airway reflexes in this patient who was recently at a bar and is likely at
risk for aspiration (i.e. alcohol, food, etc.). If this was inadequate and tourniquet deflation
was still required, I would consider employing general anesthesia by performing a rapid
sequence induction with cricoid pressure.

148
Q

The following day, the patient is complaining that his pinky finger is still numb. He
asks if something went wrong with the block in his neck. What would you say?

A

I would explain to him that, while it is possible that the residual numbness in
his pinky finger is due to nerve damage incurred during the interscalene block (or separate
ulnar nerve block), it is more likely the result of multiple contributing factors, such as
positioning, inadequate padding at the elbow, and direct pressure on the ulnar nerve from the
pneumatic tourniquet. I would also reassure him that this type of injury usually resolves
quickly(< 1 week). However, I would let him know that if his symptoms persisted beyond a
week, he should ask his surgeon or a primary care physician to arrange for an appointment
with a neurologist.

149
Q

WHAT IS CONSCIOUS sedation?

A

moderate
sedation/analgesia (“Conscious Sedation”) is a drug-induced depression of
consciousness during which patients respond purposefully to verbal commands, either
alone or accompanied by light tactile stimulation. No interventions are required to
maintain a patent airway, and spontaneous ventilation is adequate.

150
Q

What equipment would you require be immediately available during conscious sedation?

A

At minimum I would require 2 sources of oxygen, ASA standard monitors, airway
equipment, emergency medications, crash cart, a battery powered flashlight, and
personnel trained in cardiopulmonary resuscitation.

151
Q

Does ASA have same standards for whatever care they received?

A

The ASA standards for post anesthetic care apply to all locations and
all patients who have received general anesthesia, regional anesthesia, or monitored
anesthesia care. In general, the standards include transfer of the patient to a
designated P ACU or equivalent area by a member of the anesthesia team, a verbal
report of the patient given to the P ACU nurse, continual evaluation, and monitoring
in the immediate post-operative period. General medical supervision, coordination of
patient care, and discharge from the P ACU should be the responsibility of an
anesthesiologist.

152
Q

Vent settings for pt with COPD:

A

I would set his
initial tidal volumes at 6 mL/kg (ideal body weight should be used rather than actual
body weight, since lung volumes more closely correlate with height than weight).
Given the risk of auto-PEEP (air trapping) associated with COPD secondary to
limited expiratory flow (auto-PEEP increases the risk ofbarotrauma, pneumothorax,
cardiovascular compromise, and V ALI), and recognizing the increased morbidity and
mortality associated with the development of auto-PEEP in COPD patients, I would
provide adequate sedation and attempt to prolong the expiratory time by employing
low tidal volumes, a low rate of ventilation (8-12 breaths/minute), and a reduced
inspiratory time.

153
Q

Wouldn’t this strategy result in increased peak airway pressures and risk the
development of respiratory acidosis? Does this concern you? (Low TV, and no PEEP in COPD Pt)

A

A reduced inspiratory time does result in increased
inspiratory flow rate and, subsequently, increased peak airway pressures. However,
most of the peak pressure is dissipated as gas flows through the endotracheal tube and
large airways, making this an acceptable trade for an increased expiratory time, which
can significantly reduce ventilator-associated complications in COPD patients.
Moreover, the increased expiratory time leads to a reduction in end-expiratory, static
or plateau, and mean airway pressures, despite the increased inspiratory flow rate.
Finally, while hypercapnia and respiratory acidosis are not desirable, the benefits of
avoiding significant auto-PEEP most likely outweigh the potential detrimental effects
of respirator acidosis in this patient.

154
Q

How would you wean this patient from mechanical ventilation? What’s most important component?

And what is the needed respiratory support?

When would you extubatne this patient?

A

There are several options for weaning a patient from mechanically
supported ventilation such as: 1) a progressive reduction in the number of mandatory
breaths/minute while employing synchronized intermittent mandatory ventilation, 2) the incremental reduction of pressure-support ventilation, and 3) trials of total
separation of the patient from mechanical ventilation (“T-piece trials”).

However, the
most important component of separating the patient from mechanical ventilation is
the resolution of the underlying condition responsible for the needed respiratory
support.

I would consider extubation when he was awake and alert,
demonstrating active laryngeal reflexes, generating an effective cough and clearing
secretions, and when he was able to comfortably breathe without ventilatory support
for 2 hours without experiencing a deterioration of his cardiac function, mental status,
or arterial blood gasses. Moreover, I would take into consideration other proposed
extubation criteria such as: 1) a Pao2 above 60 mmHg with a Fi02 < 50%, 2) a PaC02
less than 50 mmHg, 3) an arterial pH> 7.30, 4) a vital capacity> 15 mL/kg, and 5)
required< 5 cm H20.

155
Q

Volume cycled ventilation includes: ___, ____, ____ and explain each one:

A
Volume-cycled ventilation includes assist-control (A/C), Continuous
mandatory ventilation (CMV), and synchronized intermittent mandatory
ventilation (SIMV). This mode of ventilation delivers a set tidal volume, with
airway pressures varying according to lung compliance and the selected flow rate.


o A/C- This mode is the simplest and most effective means of providing full
mechanical ventilation. In this mode, each inspiratory effort beyond the set
sensitivity threshold triggers delivery of a fixed tidal volume. In order to
ensure a desired minimum respiratory rate, a mandatory respiratory rate is
established.
o CMV-In this mode the ventilator provides mechanical breaths according
to a preset rate and volume, ignoring any patient respiratory effort. This
mode of ventilation is uncomfortable, usually requiring patient sedation.
o SIMV-Like CMV, this mode of ventilation delivers breaths at a preset
rate and volume. However, in this mode, the breaths are synchronized to
the patient’s efforts. Moreover, in contrast to A/C ventilation,patient
efforts above the set respiratory rate are unassisted.

156
Q

Pressure-cycled ventilation includes: ____, _____, ____, ____. Explain all 4

A

continuous positive airway pressure
(CP AP), pressure control ventilation (PCV), pressure support ventilation (PSV),airway pressure-release v~ntilation (APRV), a~d several noninvasive modalities.
With all of these modes, the ventilator delivers a set inspiratory pressure, with
tidal volume varying according to lung compliance. Unfortunately, changes in
respiratory system mechanics can result in unrecognized changes in minute
ventilation.

CP AP- provides a continuous level of elevated pressure to maintain
adequate oxygenation, and decrease the work of breathing. No cycling of
ventilator pressures occurs and the patient must initiate all breaths. CP AP
may be used invasively through an endotracheal tube or tracheostomy or
non-invasively with a face mask or nasal prongs.
o Pressure control ventilation- is similar to A/Cventilation, except that
each inspiratory effort beyond the set sensitivity threshold delivers a set
amount of pressure support rather than a set tidal volume. This mode of
ventilation is set to maintain a minimum respiratory rate and the preset
pressure is maintained for a fixed inspiratory time.
o Pressure support ventilation- provides ventilation support only when
triggered by the patient. Pressure is typically cut off when backpressure
causes flow to drop below a certain point. Thus, a longer or deeper
inspiratory effort by the patient results in a larger tidal volume. This mode
is commonly used to liberate patients from mechanical ventilation by
letting them assume more of the work of breathing. However, there is not
sufficient evidence to indicate that this approach is more successful.

ARPV: APRV-This mode of ventilation cycles between two different levels of
CP AP - an upper pressure level (inspiratory) and a lower pressure level
(expiratory). The bi-level positive airway pressure allows gas movement
in and out of the lung, while maintaining continuous positive pressure. It is ·
important to understand that the baseline airway pressure is the upper
CP AP level, and that this baseline pressure is intermittently “released”
(decreased) to a lower level in order to eliminate waste gas.